Conference Notes 2-12-2013

Much Thanks to Elise Lovell, MD who was the guest author of this weeks notes!
1. Trale asked about cecal vs. sigmoid volvulus. The "coffee bean" sign on abdominal Xray is useful for diagnosis in both, but in sigmoid volvulus, the bowel loop usually points to the RUQ, whereas in cecal volvulus, the bowel loop points to the LUQ. See images (look at discussion for hyperlink to additional visuals of cecal):
Attached is a review article on cecal volvulus. Sigmoid volvulus is usually seen in the elderly, especially those with chronic constipation or immobility, and those with neuropsych conditions. Cecal volvulus is less common, and can be seen in younger patients. Cecal volvulus is associated with inadequate right colon fixation during embryogenesis, leading to a mobile cecum. Prior abdominal surgery is an associated cecal volvulus risk factor. Both types of volvulus can be seen in pregnancy. Case reports have associated cecal volvulus with marathon running. Cecal volvulus clinical presentation is highly variable, whereas sigmoid volvulus patients usually present with abdominal pain, nausea, distension, constipation. Management for cecal volvulus = surgical. Management for sigmoid volvulus = reduction using sigmoidoscopy.
2. Beware of new/different headache in the pregnant/post-partum patient. They bleed, they stroke, they get hypertensive and seize. Attached is a nice article on post partum headache. There's a differential diagnosis table, as well as a table with a history driven approach to post-partum headache along with appropriate workup recommendations.
3. Finally, with regards to stab wounds to the anterior abdomen, we've reviewed some of the work of Inaba and Demetriades at USC before. Abstract from most recent big study is below. They advocate selective non-operative management of abdominal GSW as well as abdominal stabs. CT abdomen is recommended for all potentially observable GSW abdomen patients, whereas in abdominal stabs, plain Xrays and serial exams are enough if the patient is stable, no evisceration, no retained implement, no bleeding from other orifice, and no peritonitis. Like Dr. Salzman discussed today, need a low threshold for laparoscopy of left sided thoraco-abdominal penetrating trauma even in stable patients due to concern of left diaphragm injuries.
J Trauma. 2011 Feb;70(2):408-13; discussion 413-4. doi: 10.1097/TA.0b013e31820b5eb7.

Selective nonoperative management of anterior abdominal stab wounds: 1992-2008.

Source

Abstract

BACKGROUND:

The use of selective nonoperative management for anterior abdominal stab wounds has evolved into a readily accepted practice. Multiple reports have shown this strategy to be both safe and effective. However, there is a paucity of long-term studies.

METHODS:

A retrospective review was performed of all trauma patients presenting for anterior abdominal stab wounds at a Level I trauma center during a 17-year time period. Primary outcomes were the percentage of patients undergoing an exploratory laparotomy and the negative laparotomy rate.

RESULTS:

A total of 7,033 patients sustained a stab wound with 1,961 involving the anterior abdomen. The percentage of patients undergoing exploratory laparotomy decreased during the study period from 64.8% to 37.6% (overall 45.8%). The negative laparotomy rate decreased from 21.3% to 8.6% (overall 18.7%). The negative laparotomy rate of patients who underwent exploratory laparotomy immediately did not change over time (13.8%), whereas the negative laparotomy rate of those patients who underwent exploratory laparotomy in a delayed fashion decreased from 25.0% to 6.25%. The overall mortality was 1.9%, with 6.2% mortality for patients undergoing an immediate laparotomy, 0.7% for patients undergoing a delayed laparotomy, and 0.0% for patients managed nonoperatively (p<0.04). The mean length of hospital stay was 6.6 days±0.5 days, with a mean of 9.4 days±0.9 days in patients undergoing an immediate laparotomy, 8.1 days±0.5 days in patients undergo a delayed laparotomy, and 3.8 days±0.2 days in patients managed nonoperatively (p<0.001).

CONCLUSIONS:

Selective nonoperative management for stab wounds to the anterior abdomen is associated with a decreased operative rate and decreased negative laparotomy rate over time. Selective nonoperative management is both safe and effective for anterior abdominal stab wounds.

Conference Notes 12-18-2012

Conference Notes 12-18-2012

Last Conference until  1-8-2013.  You can use that time to donate to our EM Foundation to benefit resident education/development.  Thanks and Happy Holidays!
/em-foundation/

Albanis   Ophtho Emergencies

Three key questions: Vision Loss?   Pain in the eye?  Previous Surgery?    Any previous hx of eye surgery puts pt at higher risk for endophthalmitis  or globe rupture.   Both can occur years after eye surgery.  The risk is probably highest following glaucoma surgery.

Visual acuity is the most important thing you evaluate about the eye.   VA is prognostic.  Better VA’s are more likely to do better than patients with worse VA’s for a similar process.

Barounis question: How do you get lids open with a swollen eye?  Albanis and Harwood comments: You need lid retractors or use bent paperclips to retract lids.   Check for 4 basic things: Is the globe grossly intact?   Does the pupil react?   Is there a hyphema?  Is there a 360degree sub conjunctival hemorrhage?  If these 4 are OK then you can move on, there is unlikely to be a severe eye injury.  If any are abnormal you should consult/image.

Case 1: Amaurosis Fugax: painless monocular vision loss, due to an embolism to the retinal artery or a branch thereof.

Stye is an acute staph infection.  Treat with warm compresses/lid scrubs/optional oral antibiotics if you have concerns about a preseptal cellulitis developing.  For styes that are really pointing resist the temptation to incise it with a needle. The ophthalmologists sit on these using warm soaks/lid cleansing/possible abx for 4 weeks prior to surgical treatment.  Concerns about surgery are that it can cause scarring or affect the mebomiun glands.  Chalazion is basically treated the same as stye.

Eyelid lacerations:  ER docs stay away from repairs near the medial canthus, and probably don’t repair lacs that involve lid margins.   Both are high risk for complications.  The general opthomologists don’t even repair lacs around the medial canthus; they refer to oculo-plastics.

Great test to identify myasthenia gravis in a kid with ptosis: Place an ice pack for a couple of minutes on the ptotic eyelid.  When you take the ice pack off and sit the patient up, if they have MG, the ptosis will temporarily resolve.

Subconjunctival hemorrhage in the setting of non-trauma is virtually always benign even if it is 360 degrees.    In the setting of trauma, a 270-360 degree subconjunctival hemorrhage is suspicious for a globe rupture.  If there is any second finding (vision loss/hyphema/altered pupil/etc)in association with this severe subconj hemorrhage, patient will likely go to surgery.   CT can help in this situation.

Barounis/Chastain    STEMI Conference

CASE 1: Posterior MI: Diagnosis is difficult because no specific leads represent this area.   You need posterior leads to verify the infarct (post leads are specific but not sensitive).   ST segment depression V1-3.  Prominent R wave V1-3.  Posterior MI frequently co-exists with acute inferior or lateral MI.  The vessel commonly involved in an isolated posterior MI is the circumflex.   If you are uncertain whether patient has an AMI, stat 2D Echo can help identify wall motion abnormality.   Harwood comment:  this patient had an isolated circ lesion causing isolated posterior mi.   Pt partially reperfused his vessel with his own TPA accounting for his improved 2nd EKG and temporized his clinical status until he was taken to the cath lab.

Case 2:  75 yo male with chest pain and new LBBB.  Harwood comment: pt has new LBBB and new first degree AV block putting him at risk for complete AV block.    Comment from care coordinator: If you document new LBBB on the first EKG in the chart, that case goes into the STEMI care bundle.  If you don’t feel the patient has a STEMI you need to document why the patient is not going to cath lab.    The STEMI Care bundles don’t factor in Sgarbossa criteria.    Proposed management for suspected ACS with presumed new LBBB: if unstable or CHF go to cath lab.  If stable with Sgarbossa criteria present, go to cath lab.  If Stable with no sgarbossa criteria present get stat echo.  Simplified Sgarbossa criteria are concordant st elevation >1mm or concordant st depression V1-3 >1mm,   1 positive Sgarbossa criterion is 95% specific but not sensitive.   The discordant ST elevation of 5+mm anteriorly has been taken out because of a relatively low likelihood ratio.

Case 3: Pt had a right coronary artery lesion.  He had waxing and waning perfusion resulting in intermittent accelerated idioventricular  reperfusion rhythm.  

Barouinis/Chastian/Harwood  Flash Mob Journal Club     

Article discussing oxygen toxicity due to post arrest resuscitation.

Background: Therapuetic hypothermia post-arrest is thought to prevent reperfusion oxidative injury.  Patients have some brain edema post arrest.  High oxygen exposure to the brain  during resuscitation is thought to increase oxidative injury by the production of excess free radicals.

Methods: 173 patients treated with therapeutic hypothermia.    Retrospectively the investigators identified the highest PAO2 recorded for each patient in the first 24 hours post arrest.   They correlated this PaO2 measurement with survival and  functional neuro outcome.    All patients were in the target therapeutic temperature range.

Outcome: Survivors had lower PaO2 compared with nonsurvivors.   Higher PaO2 associated with worse neuro outcome.   Cut off seems to be a PaO2 of220.   Above that level patients did progressively worse with higher PaO2 level.

Potential confounders:  High PaO2 could be just a surrogate marker for a sicker patient or poorer quality of the resuscitation.   The resuscitation team may have had to resuscitate the patient more aggressively and were hesitant to dial down the O2 or the resuscitation team may be subpar and were not watching the oxygenation level closely to optimize patient care.

What to do: The emergency physician may want to closely watch oxygenation and avoid hyper-oxygenation in the post arrest patient.  Consider  starting your vent settings for the post arrest patient at 50% FIO2.   Harwood comment: think of oxygen as a medication and there is a correct dose of oxygen for the post-arrest patient.  Giving boat loads of oxygen after resuscitation and damaging a patient’s brain is analogous to giving too much tobramycin and bagging a patient’s kidneys.    

Animated discussion started by Sola’s question and response lead by Harwood and Brian Febbo about supplemental oxygen in other types of patients.    No benefit of supplemental oxygen above 96% room air sat for  sickle cell patients.   Cochrane review shows harm to providing supplemental oxygen to acute MI patients with room air sats over 96%.

Badillo   Chest U/S

Ultrasonographic A lines are naturally occurring artifact lines or air lines.  They are horizontal on the screen and are stacked on each other.    B lines are bad lines and are signs of interstitial edema.  They are vertical  and extend down to bottom of screen.  3 or more B lines are consider pathologic.  The more you see the more specific the finding.

Parasternal long view:  Kinesis assessment by looking at whether the mitral valve anterior leaflet hits the septum and do the walls of the LV almost come in contact in systole.

Case1:  CHF patient with poor kinesis on echo and B line comet tails that signify pulmonary edema.

Case 2: US is better than supine CXR for identifying pneumothorax.  Use the linear probe in midclavicular line in area of 2nd intercostals space.   Look for sliding pleura between rib shadows.  There may be small B lines(also called minor B lines, comet tails, Z lines)  that don’t go all the way down to the base of the screen and are normal artifact as opposed to the abnormal B-lines that signify CHF.    There is an M-mode view that can also identify pneumothorax.  Patients with a pneumothorax have a “barcode” type appearance.

Big RV with bowing of septal wall toward LV raises concerns for PE.

Practical Scanning with the Residents.

Applicant Review

Conference Notes 12-11-2012

Please consider a gift to our EM Foundation to benefit resident education/development.  Thanks!
/em-foundation/

 

Conference Notes 12-11-2012

Walchuk/Watts   Oral Boards

Case 1 Esophogeal variceal bleeding:  Fluid/blood resuscitation.  Manage airway as patient becomes confused.  Protonix drip.  Octreotide 50microgram bolus followed by 25-50mics/hr.  Consult GI.  Could consider IR for TIPs procedure.   Correct any coagulation defect with FFP or Factor 7A. Give rocephin or cipro to avoid infectious complications and limit re-bleeding.   Endoscopy with sclerotherapy or banding teamed up with Octretide has better outcome than either endoscopy or octreotide alone.

Case 2 Organophosphate Poisoning: IV hydration, decontaminate, give Atropine and 2-PAM.   Usually need large doses of atropine. SLUDGE BBB=salivation/lacrimation/urination/diarrhea/gastric emptying/bronchospasm/bronchorhea.  Usually due to insecticide poisoning.  Most common pitfall is inadequate atropinization.

Case 3 Roseola infantum:  Also known as sixth’s disease/exanthema subitum/three day fever.  Patient is not contagious at the time of rash appearance.   Non-pruritic Rash appears after 3-5 days of fever.  Pt is well appearing.      Erythema infectiousum (fifth’s disease) will have slapped cheeks and lacey extremity rash.  It is usually in school age children.

Elise comment: Utilize massive transfusion protocol for GI bleeders in shock.  Could also consider FEIBA for coagulopathy.  Get 2 IV’s or a cordis in these patients to give large volume crystalloid/blood product replacement.   Any tox case on oral boards call poison control.   Figure out the OD by talking to family/friends/pre-hospital personnel.

Harwood comment:  Start to mobilize atropine 10-50 amps for treatment of organophosphate poisoning as soon as you suspect the diagnosis.   You will burn through all the atropine in the ED.  

Ketanneh  Trauma

The NEXUS decision instrument stipulates that radiography is not necessary if patients satisfy ALL five of the following low-risk criteria:

  • §  Absence of posterior midline cervical tenderness
  • §  Normal level of alertness
  • §  No evidence of intoxication
  • §  No abnormal neurologic findings
  • §  No painful distracting injuries

Insignificant injuries were defined as those that would not lead to any consequences if left undiagnosed. The NEXUS investigators evaluated 34,069 blunt trauma patients who underwent radiography of the cervical spine comprised of either a three-view cervical spine x-ray or a cervical spine computed tomography (CT) scan. Of these patients, 818 (2.4 percent) had sustained a cervical spinal column injury. Sensitivity, specificity, and negative predictive value (NPV) of the NLC were found to be 99.6 percent (95% CI 98.6-100), 12.9 percent (95% CI 12.8-13.0), and 99.9 percent (95% CI 99.8-100), respectively. (These notes cut/pasted from Up to Date)

Tenderness should have objective signs like physical evidence of discomfort.

Altered level of alertness: GCS<15, disorientation, Inability to remember 3 objects at 5 minutes.

Painful distracting injury: Undefined and varies by physician. 

 Canadian C-Spine rule

The CCR involves the following steps:

  • §  Condition One: Perform radiography in patients with any of the following:
    • ·         Age 65 years or older
    • ·         Dangerous mechanism of injury: fall from 1 m (3 ft) or five stairs; axial load to the head, such as diving accident; motor vehicle crash at high speed (>100 km/hour [>62 mph]); motorized recreational vehicle accident; ejection from a vehicle; bicycle collision with an immovable object, such as tree or parked car
    • ·         Paresthesias in the extremities
  • § 
    • ·         Simple rear end motor vehicle accident; excludes: pushed into oncoming traffic; hit by bus or large truck; rollover; hit by high speed (>100 km/hour [>62 mph]) vehicle
    • ·         Sitting position in emergency department
    • ·         Ambulatory at any time
    • ·         Delayed onset of neck pain
    • ·         Absence of midline cervical spine tenderness

Patients who do not exhibit any of the low-risk factors listed here are NOT suitable for range of motion testing and must be assessed with radiographs.

If a patient does exhibit any of the low-risk factors, perform range of motion testing, as described in Condition Three below.

  • §  not

In the derivation study, the CCR demonstrated a sensitivity of 100 percent and a specificity of 42.5 percent for identifying clinically important cervical spine injuries [22]. In 2003, the CCR was prospectively studied in the emergency departments of nine Canadian tertiary care hospitals. Of 8283 patients, 162 were found to have clinically significant injuries, and the sensitivity, specificity, and negative predictive values of the CCR were respectively 99.4 percent (95% CI 96-100), 45.1 percent (95% CI 44-46), and 100 percent [23]. The investigators reported that the CCR would have missed one patient with a clinically important cervical spine injury, while the NLC would have missed 16. The CCR has also been validated in larger hospital-based studies and in an out-of-hospital study of paramedics [24,25].  (These Notes cut and pasted from Up to Date)

Negative CT Cspine with persistent pain:  3 management options: continue collar with f/u with neurosurg.  Get MRI if neg then remove collar.  Get the dreaded flex-ext cspine films.   The utility of Flex/ext films is controversial.

Negative CT Cspine and Obtunded:  Don’t do flex/ext films.  Do MRI.   One study found 9% rate of abnormal MRI in obtunded patients with negative CT Cspine.   If you need to leave patient in collar for a prolonged time, use an Aspen Collar.

EAST guidelines were discussed and are basically a combination of NEXUS/Canadian rules with the above recommendations for what to do with the patient with negative CT cspine and continued concern for injury.  (If you want more info you can find the EAST guidelines at east.org)

 
 
 
 
 
 
 

Girzadas/C. Kulstad/Harwood comment:  Agreement  of commenters that there is still some utility to do a Cspine plain film series in a low risk/young patient.  You avoid excess radiation exposure and still can see C1 to C7. In a patient with higher risk or any increased difficulty in getting good quality plain films go right to CT Cspine.   Dr. Smith (Trauma) felt strongly you need a CT to rule out Cspine fracture.

Barounis comment: If you are working in a community ED and are transferring a severely injured patient that is obtunded, get a cspine film/cxr/pelvis xray.

Harwood/Smith comment: Don’t do flex/ext films.  Go right to MRI in the patient with significant neck pain or obtunded and negative CT Cspine.

Gottesman  M and M

25yo male with GSW to right face.  CT head showed facial fractures.  No intracranial injury. No cspine injury on CT Cspine

Lefort Fractures:  Type 1 fracture line goes through alveolar ridge and allows the front teeth to move with traction.    Type 2 fractures allow the nose to move.  Type 3 fractures allow the zygomas to move.

Patient in case became agitated in CT and developed hypoxia.  Trauma team moves to attempting intubation.   Glidescope intubation was successful.

Avoid nasal intubation in patients with facial fractures.

Pt developed increased bleeding from facial wound.  Wound/nose/oral cavity were all packed.

Causes of agitation: Head injurying/toxins/infection/hypoxia/hypoglycemia/intracranial injury/stroke/seizures/emotional stress/sensory deprivation.  Psychiatric cause is basically a diagnosis of exclusion.

Pt was found to have a high Internal Carotid Injury on CTA.  Went to OR but pt did not have surgery because he had an injury that the surgeons could not get to.   His injury was zone 3.

Harwood comment:  sometimes these high zone 3 injuries require surgical removal of mandible during the time of surgery.  Patients that have this procedure frequently have difficulty with eating and talking the rest of their lives.

Elise and Harwood comment: If you see this patient in a community ED, Tube/get facial film/cross table lateral cspine film/cxr prior to transfer.  

Patient in Case blows a pupil.  Pt stroked from left internal carotid injury and developed herniation

Take home point: Beware the agitated patient.  Think of hypoxia and other serious causes prior to attributing to psych reasons.

Harwood comment: As soon as you decided this patient has to go to CT you should have intubated him.

Garrett-Hauser     Ethics   Mandatory Reporting

Case 1:  Female patient with AIDs was having sexual relations with a man who did not know her HIV status.  They use condoms during intercourse.   Do we have obligation to notify her partner?

There is a law on the Illinois’ books titled Criminal Transmission of HIV.  Person has to know they have HIV and have sex without a condom with a person who does not know they have HIV.   Prosecution requires a court order for release of medical records.   So we don’t need to notify the police.     Harwood comment: If you know that HIV person knowingly    has sex with partner with no condom, do you notify the non-HIV partner?   He would first try to convince the HIV patient to notify the non-HIV patient.  Dr. Garrett-Hauser made a strong point that it likely would be a HIPPA violation to call the police.  There was a animated discussion among residents and attendings about whether and  how to notify the partner at risk.   Elise and Harwood would notify the at-risk partner.   C. Kulstand and Girzadas would give the at-risk partner a more obtuse warning such as: As a physician, I would advise you not to have sexual relations with that person until you have a detailed conversation with that person.

Case2: Intoxicated parents bring child to ER. Don’t let parents drive the child home.  Notify DCFS.

Case 3: 14yo female found to be pregnant at PMD’s office.  Pt’s mom brings child to ER that same day for U/S and was threatening lawsuit because the 14yo was in ER one week ago and diagnosed with pregnancy  and mom was not told.   ER records from previous ER visit documents that 14 yo patient refused to allow staff to notify mom of her pregnancy.     Shayla discussed that the decision to notify parents of medical condition of a minor is a balance of patient’s cognitive maturity and risk of medical condition.   The state law for emancipated minors specifically gives emancipated minors the right to consent for treatment  but not specifically the right to privacy.   Discussion of risk came down to: the parent is much more likely to sue you for not notifying them than the young patient is to sue you for ratting them out.

Case 3: Illinois does not have a mandatory reporting law for domestic violence.  The only state with such a law is California.  In California, ED physicians are mandated immediate reporters.  These laws are controversial because there has been increased morbidity or mortality for patients in which MD’s reported before the patients were personally ready to have abuse reported.

RLT Applicant Review

Procedural Tip: when performing a digital block, wait 10-15 minutes to optimize anesthesia.  Tell the patient you are going to see another patient and will be back in a few minutes. Don’t leave them too long though.  It’s embarrassing to have to anesthetize them twice.   I like to use bupivicaine without epi because it lasts longer than lido. 

Conference Notes 11-27-2012

Conference Notes 11-27-2012

Putman/Coghlan    Oral Boards

Case 1: Pt with TIA 8 days after AMI.  Pt had a persistent ventricular aneurysm with likely mural thrombus.   Dr. Coghlan said heparin would be indicated for mural thrombus.   Mural thrombus develops in the first week after AMI and embolizes in the second week. 

Case2: Child with sickle cell disease and splenic sequestration crisis.  Have to assess for splenic rupture. Give IV fluids and transfuse 10ml/kg PRBC’s.   Mortality in splenic sequestration crisis is 12%.

Case3: Child swallowed a button battery.  Battery was identified in stomach. Management was to observe as outpt with serial xrays.  

Elise comments: Fluids and O2 in sickle cell is no longer universally indicated.  If spleen is enlarged you know you will need a blood transfusion.   NL systolic BP in Kids is 2 X age plus 70.   If button battery is ingested with magnets it increases risk of perforation. 

Harwood comment: If button battery is not making progress on serial xrays, the kid will need a scope.  Transfusing blood in kids with shock you can give 20ml/kg of PRBC’s.

Coghlan comment: Sharp objects such as needles need to be removed.

Group discussion about heparin for TIA with possible mural thrombus.  Everyone agreed that this was a difficult/ controversial situation.  You would like to have an echo and have neuro and cardiology consults to help make this decision.

RLT/Residents  Applicant Review

Discussion of the progress of recruiting season.

Schroeder  Visual Diagnosis in Pediatric Patients

Paronychia with lymphangitis.  Lymphangits is associated with Group A strep infection.  In young kids, admit for iv antibiotics.  Older, well appearing kids can go home on po antibiotics.

Mucocoele on inner lip.  Blockage of minor salivary gland due to biting lip.  Can refer for excision.

Mastoiditis: Trend to do less ct’s and manage clinically with antibiotics.

Salmon colored debris in diaper is urate crystals.  Young kids excrete more urate crystals. It is benign. Not associated with infection or other problems.  

Thick white exudate on a 2yo tongue=thrush.  Could be due to inhaled steroids.  Could be due to having child sleep with bottle in the mouth.

Erythema migrans from lyme disease.  Circular erythematous rash. Pt had tick in hair.  Lyme disease is increasingly found in Illinois counties.

Herpetic whitlow can be complicated by Group a strep causing lymphangitis.  Herpetic whitlow also uncommonly can have signs of lymphangitis due to the virus itself.  Harwood comment: I would treat with both anti-strep antibiotic and antiviral medication.

Eruption hematoma is basically a blood blister associated with an erupting tooth.  Nothing to do about it.

Kids can have a sucking blister on their lips from sucking on their lips.  It can look like cyanosis in darker skinned children.

Bed bug bites result in a rash that is nonspecific.  With itchy macular-papular rashes think scabies and bed bugs.   Families with bed bug infestations need to consult a professional company. 

Aphthous ulcers are caused by T-cell immunity.   Treat with mouthwash rinses or oral steroid gel.

Cobblestoning of the posterior pharynx can be due to post nasal drip.  Treat with nasal steroid or Sudafed.

Bullous impetigo can be treated with mupirocin topically.  Can treat more extensive infections orally with bactrim and keflex.  There have been MRSA strains of impetigo isolated.  Treat all impetigo as potential MRSA infection.  Mupirocin covers MRSA.  If using oral abx: bactrim/keflex or clinda to cover both MRSA and strep. 

Eczema herpeticum is a disseminated HSV infection.  Most often in kids with bad eczema. Treat with IV acyclovir.    Also give iv antibiotics  for possible severe impetigo until cultures clarify diagnosis.  You can use the new cultures we have in the ED for both bacterial and viral cultures.

Unilateral parotid swelling is more likely to be bacterial so treat with abx.  Bilateral swelling is more likely viral.   Harwood comment: with parotid swelling from infection the ipsilateral ear will pop out somewhat like mastoiditis.  This can be a clue to help you localize the swelling to the parotid.

Bill showed pictures of hand foot mouth.  The vesicles with HFM are thick walled, flat and grey compared to chickenpox vesicles.   Also Chicken pox won’t affect the palms and soles.  HFM can have lesions on more proximal extremities and face in addition to hands and feet.  There are seasonal variations to the severity of HFM.  In Cambodia there was a fatal form of HFM.  Bill felt that fatal cases have been localized to Asia and we should not alter our practice of discharging most patients in the US at this time.

Kerwin  Atrial Fibrillation

Who gets afib: age, htn, coronary heart disease, valvular disease, chf, HCM, congenital heart disease, obesity, dm, thyroid disease, CKD, cardiac surgery,  family hx, ETOH (holiday heart syndrome), PE, sleep apnea, numerous medications cause.

A-fib increases risk of all-cause mortality, CHF, and stroke.  

Paroxysmal AFIB lasts less than 7 days.  Persistent AFIB  last longer than 7 days. Permanent AFIB is afib in which attempts at cardioversion failed.   Lone afib is in patients <60 with no cardiopulmonary disease including HTN.

Get usual labs and TSH to work up. Get troponin as part of  work up only if ischemia is suspected.

NEJM study 2002 showed no survival advantage of rhythm control vs. rate control. Multiple other studies confirm this finding.  But in 2012 study in Circulation showed decreased rate of stroke in rhythm control patients.

ED management: Divide patients into 3 categories.  Pts who are unstable.  Pts with afib less than 48 hours.  Pts with afib more than 48 hours.  

Afib >48 hours: Rate control and anticoagulation are management goals.  No clear data on superiority of beta-blocker or calcium channel blockers for rate control.   No benefit from strict rate control (<80) vs. lenient rate control (<110).   For patients with heart failure consider digoxin or amiodarone instead of beta blocker/ca channel blockers.

CHADS2: CHF, HTN, Age>75, DM, Prior stroke or TIA. This tool assesses stroke risk with afib.  With a CHADS2 score of  0 or 1 you can opt to treat with ASA and not warfarin. 

Hemodynamically unstable patient: Cardiovert.  Anterior-posterior pad positioning is optimal.  Higher energy doses are more effective.  Cardioversion does not cause myocardial injury so higher dose is not really a concern.  If you have to shock more than once, wait at least a minute between shocks.  Consider giving push dose pressors to support BP prior to cardioversion.  1ml of phenylepherine in a 100ml bag of saline.  Mix and draw up 10ml of this solution. Can give 1-2 ml every 5 minutes.  

Afib<48 hours: Can attempt rhythm control. It avoids admission and meds.    Ottawa aggressive protocol includes procainamide 1gm over 60 minutes.  If that doesn’t work sedate and cardiovert with 150j. Converted patients discharged on no meds.  Multiple studies seem to show No increased risk of death or stroke from early cardioversion protocol.  About 8% of patients had 7 day recurrence.   Another consideration is that the rate of spontaneous conversion in new onset afib patients is up to 70%!  So centers will send patients home and ask them to return in 24 hours for a recheck.

Afib with WPW use cardioversion if unstable. Use procainamide if stable.

Afib in pregnancy: CCB’s , Beta-blockers, and digoxin are all ok. Cardioversion is ok if unstable. IV heparin is ok.

Harwood comment: Treating unstable afib patients use highest dose possible of cardioversion.  If not effective, wait a minute or two and try again.   If second shock is ineffective, give procainamide over 30 minutes and retry shocking.  Get cardiologyto help you if the first two shocks don’t work.  If you need to support BP give push dose pressors or pressors on drip.

Barounis comment: How do you manage the septic patient with afib?  Harwood responded to give digoxin.  Shock if necessary but most of these patients are in afib chronically.  Other attending suggested IV magnesium or amiodarone as alternative approaches.

Conference Notes 11-20-2012

Conference Notes   11-20-2012

Chandra    Thoracic Aortic Dissection

Tupak Shakur is still alive! (Urban myth)

Stanford Type A involves ascending aorta.  Type B Stanford is only descending aorta.

Two different anatomic systems, the DeBakey and Daily (Stanford) systems, have been used to classify aortic dissection [2,23-25]. The Stanford system is more widely used. It classifies dissections that involve the ascending aorta as type A, regardless of the site of the primary intimal tear, and all other dissections as type B. In comparison, the DeBakey system is based upon the site of origin with type 1 originating in the ascending aorta and propagating to at least the aortic arch, type 2 originating in and confined to the ascending aorta, and type 3 originating in the descending aorta and extending distally or proximally.

Ascending aortic dissections are almost twice as common as descending dissections. The right lateral wall of the ascending aorta is the most common site of aortic dissection [1]. In patients with an ascending aortic

 

6.4% of patients have been found to have painless aortic dissection.  Most of the time these are ascending dissections that present with stroke or syncope.

In patients with shock in the setting of suspected aortic dissection, think pericardial tamponade.  The dissection can bleed into the pericardial sac.

Diagnosis:  Ekg is mostly non-specific but can have ami in 5% of patients with dissection.  CXR can show wide mediastinum.  CT angio has sens/spec in the mid to high 90%’s.   MRI has the highest sens/spec 98%!  Aortography has sensitivity of 88% and specificity 94% so it is no longer used.  TEE has a sens/spec of about 98%.   This is probably the way to go for the unstable patient or the patient with high risk for aortic dissection who can’t take contrast.  Harwood comment: If you aren’t sure if the patient has a PE or dissection, get the CT PE protocol because it is harder to identify PE than dissection.  Harwood and Elise both agreed that if you suspect one diagnosis over the other, get the test specific for that diagnosis.   The timing of the dye load is different for both tests and if you get the CT PE study it decreases your sensitivity for dissection.  The CT test for dissection lowers your sensitivity for pe.  

Management:  BP control start with esmolol and follow with nitroprusside.  You want to decrease shear stress on the aortic wall.   Labetalol is another good single angent choice.   Maletich comment: How bout esmolol and nicardipine?  Paarul/Harwood/Elise al felt this was reasonable as well.   Type A dissections are managed surgically.  Type B dissections are managed medically unless they have signs of ischemia, progressive dissection, intractable pain,impending rupture, or uncontrolled hypertension.   Harwood comment: You have to advocate for the patient and get them to the OR with descending dissections when they have one of the above complications.

Poor prognosis: advanced age, hypotension, renal failure, pulse deficit, need for massive transfusion, many others.

Girzadas comment: Some respected authorities say It is the standard of care to miss aortic dissection.  Harwood comment: Aortic dissection is the hardest diagnosis in the world.   Harwood’s own decision rule for dissections is classic aortic pain get ct angio.  If pain is not classic but you still have suspicion: check bp’s in both arms if abnormal get a CT angio,  if chest pain and abnormal cxr get CT angio,  if chest pain and abnormal d-dimer get a ct angio.   Coghlan comment: remember this diagnosis in pregnant patients and patients who had a recent coronary angio.   Gupta comment: this diagnosis is rare and difficult. It should be considered ok to miss this diagnosis.   Work up for dissection delays diagnosis of other more common diagnoses and these patients generally do poorly.   Chastain comment: gotta look at your own CXR’s because you have a different perspective on the mediastinum than the radiologist.

Putman     Neck Trauma

 

 

Harwood comment: Intubate early to avoid losing the airway.   The risk of losing airway due to expanding hematoma is much greater than an early intubation.   Doing a cric in a patient with neck trauma is very difficult.    Lovell comment: You have one shot to get this airway.  Go right to most reliable device which may be glidescope.

Prior to RSI, check for neuro deficits especially cranial nerves.   You only need to immobilize cspine in patients with blunt trauma and/or neuro deficit.   Isolated penetrating neck injury does not require cspine immobilization.

Penetrating Trauma Hard signs: hypotension, arterial bleed, decreased carotid pulse, expanding hematoma, thrill or bruit, lateralizing signs, air bubbles in wound, hemoptysis/hemetemesis, tracheal deviation.  These all go to the OR.

Penetrating Trauma Soft signs: hypotension in field, history of arterial bleed, bradycardia, large hematoma, apical cap on CXR,  stridor, hoarse voice, subQ air,  CN7 deficit.  These patients all require further evaluation.

Zone 3 injuries need CT head.  

Helical CTA is the work horse study for neck injuries.  It shows vascular structures as well as cspine and aerodigestive tract.    Harwood comment: Selective angiography is very rarely done.  CTA is the way to go all the time, unless there is metallic FB’s or hardware obscuring the CTA image.

If CTA is neg then patients should get CXR, EGD and contrast esophogram.  Also they get laryngoscopy/broncoscopy.    Zone 3 injury gets catheter angiography.

Blunt Trauma

Wallenberg syndrome:  Can get from vertebral artery injury

Common Symptoms

  

 

Ipsilateral facial pain and numbness

  

 

Vertigo, nausea, and vomiting

  

 

Ipsilateral clumsiness

  

 

Diplopia, oscillopsia

  

 

Numbness ipsilateral or contralateral to lesion

  

 

Dysphagia, hoarseness

Signs

  

 

Contralateral:

  

 

Impaired pain sensation over half the body, sometimes including the face

  

 

Ipsilateral:

  

 

Impaired sensation over half the face

  

 

Ataxia of limbs, falling to side of lesion

  

 

Horner syndrome

  

 

Dysphagia, hoarseness, paralysis of vocal cords

  

 

Diminished gag reflex

  

 

Loss of taste

  

 

Other:

  

 

Nystagmus:

  

 

Primary-position rotatory

  

 

Gaze-evoked horizontal

  

 

Downbeating on lateral gaze

  

 

Ocular skew deviation

  

 

Hiccup

 

E Kulstad   CV Study Guide

Asa has 4% absolute mortality benefit from 13% to 9% in AMI (25% relative risk reduction).   ACMC’s overall AMI mortality is 15.2% which is lower than the national norm.

Bening Early Repol: widespread ST elevation (precordial greater than limb leads), J point elevation, concave up st segment, notching of J point,  prominent concordant t waves.   Girzadas comment: BER should not have reciprocal st depression

Electrical alternans is a sign of pericardial effusion.  Low sensitivity/high specificity.

 

Electrical alternans   Every other QRS complex has a reduced amplitude alternating with a higher amplitude

 Stress testing:  Sensitivity/specificity is relatively low for coronary stenosis.  Post-test probability = pretest probability X LR.  The LR for stress testing is 0.25.  So if you have a patient with 10% risk of coronary stenosis and he has a negative stress test, you bring his risk down to 2.5%. It is debatable about whether this is low enough for feel comfortable with.

Anterior wall AMI with new LBBB and first degree AV block: Pt should get stand by prophylactic temporary pacemaker.  (Zoll Pacer Pads)   

Plavix has not been shown to significantly decrease mortality  in ACS.  We use it because the composite end point of death/stroke/non-fatal mi is lower than placebo.  This outcome has been accepted by our cardiology colleagues as beneficial for patients.

Heparin has also not been shown to lower mortality in ACS compared to placebo in a Cochrane review. Heparin has been shown to lower the risk of subsequent AMI in ACS.  Cochrane review showed no difference between heparin and LMWH.  As of today amazingly, the Cochrane review has not been concluded on heparin for STEMI.

History of CHF is a marker for risk of sudden death following syncope.

Beta-blockers and ACE-I’s and spironolactone have been shown to reduce mortality in CHF.  Lasix has not been shown to decrease mortality.

High dose IV NTG has been shown to improve outcomes in acute CHF exacerbations.

CHF mortality is 50% in 4 years. 

Contraindications to TPA in AMI:  Prior ICH/Intracranial malignant tumor/structural cerebral vascular lesion/ischemic stroke in last 3 months/significant closed head trauma in last 3 months/active bleeding or bleeding diathesis.

Smoking cessation has the best mortality benefit of any intervention in CHF.

Sgarbossa Criteria:

The three ECG criteria with an independent value in the diagnosis of acute infarction and the score for each were:

  • § 
  • § 
  • § 

A minimal score of 3 was required for a specificity of 90 percent. The first two criteria are similar to those described above since the ST segment is concordant rather than discordant with the QRS complex. However, the third finding requires further validation, since a high take-off of the ST segment in leads V1 to V3 has been described with uncomplicated LBBB, particularly if there is underlying left ventricular hypertrophy. In a substudy from the ASSENT 2 and 3 trials, the third criteria added little diagnostic or prognostic value [15].  Harwood comment: It is not helpful in the vast majority of patients with LBBB. But when you have it is pretty specific.

Modi  LVADS

Who can receive LVAD:  Class 4 heart failure on max medical therapy, people who can’t come off bypass, cardiogenic shock, bridge to transplant.

Only 24 heart transplants in Illinois this year.  112 patients on destination therapy LVADS.  Longest living patient on LVAD at ACMC is 8 years.

LVAD patients do not have a pulse. The pump is not a pulsatile pump, it is continuous flow.  Probably more directly a cause is that the aortic valve doesn’t open with this pump.  The pump  connects the LV to the aorta.  There may be more intracranial hemorrhages due to continuous laminar flow.   Newest pump has a magnetically levitated internal rotor device so there is no wear on the rotor internally.

Initial eval: check for fever.  Patients are at risk for infection with drive tube in abdomen.  Most patients will not have a pulse. No affect on respirations.  Blood pressure is usually around systolic of 90-120. Ekg is unaffected by LVAD.   Pt’s may have VT while on the LVAD but will still have the pump working.  Patients will be alert and talking with VT.  You don’t need to shock patient if they are alert and talking and not in shock.  Treat the arrhythmia with meds until pt has altered mental status/dyspnea/low bp.  When evaluating a patient, check the computer reading, it will likely tell you of any problems with the device.   If pt is unresponsive and arrives in the ED with a disconnected drive line, hook it back up.  You run the risk of throwing a clot from the heart but the alternative is death for the patient.   Chest compressions are ok for unresponsive patients with LVADs.  Check the LVAD and make sure it has power.   Otherwise perform resuscitation as you normally would. 

Lovell Population Health

Population health aims to improve the health of a population.  Reduces healthcare inequities.  Aims to improve the overall health environment of a population.

Accountable Care Organizations and similar models focus on quality, preventative medicine approaches and shared savings instead of trying to increase the volume of healthcare delivered.   Avoid unnecessary care and avoiding complications.  We are accountable for all the patients in our catchment area. 

This will be a new paradigm that we will be part of moving forward in our careers.

Erickson   Eye Emergencies

Find out if the patient uses glasses or contacts.

Get a visual acuity of the affected eye!!  Vital sign of the eye. 

Chemical injury:  Irrigate the heck out of the eye.  Use topical anesthetic prior to irrigation. Check ph pre and post irrigation.

Acid causes coagulation necrosis.  Coagulation necrosis actually prevents deeper penetration of acid into tissue.

Alkali causes liquefaction necrosis which is more serious and can penetrate more deeply into tissue.

Acute angle closure glaucoma: Precipitated by papillary dilation.  Eye pain and emesis/headache.  Hazy cornea and fixed mid-dilated pupil.   Tx with beta blocker, alpha agonist, pilocarpine and mannitol and acetazolamide.  Mnemonic: E=m2c:  save the eye=miotics (pilocarpine/alpha agonist/beta blocker) and mannitol,  carbonic anhydrase inhibitor.

Ruptured globe: eye pain, decreased visual acuity, teardrop shaped pupil, positive sidel’s test. CT orbits can be helpful.  CT may show flat tire sign.

Corneal abrasion: topical antibiotics, consider a cycloplegic agent.  Don’t give them topical anesthetic for home.  Update tetanus shot.

Linsey went through a number of pattern recognition slides of eye emergencies.

Conference Notes 11-13-2012

 

Conference Notes 11-13-2012

Chastain /Marra  ENT Emergencies

FB in the Ear: Enough light is critical to seeing what is going on in the ear.   Otoscope lights are generally inadequate.  Amplify your light with a head lamp.   Dr. Marra prefers using a blunt right angle device to remove ear FB’s.   If you touch the ear canal or tm you will lose any cooperation of the patient.  The skin of the ear canal is directly adherent to the ear canal bone.  Any shearing force will cause hematoma or bleeding.   Dr. Marra stated that for ear fb’s you may be better off using a nasal speculum in the external ear canal and a head lamp than using an otoscope.  He felt that if you need to sedate a child to remove a FB, you probably should be referring the case to him at that point.  He said you don’t want to sedate the child and then not get the FB.   When he sees the child in his office he will use a microscope and a blunt right angle tool to attempt removal. If he is unable to remove the FB he will take child to OR for anesthesia.   If a patient has a live bug in the ear he will drown the bug with mineral oil.   He doesn’t use lidocaine to kill the bug because if the patient gets a TM perforation they could develop severe nausea from the lido.

Epistaxis: First attempt to visualize the nasal septum to see If bleeding is anterior.  Neosporin topically is more effective than Vaseline in kids because kids’ noses are frequently colonized with staph.  Staph is frequently a cause of recurrent epistaxis in kids.  Dr. Marra uses silver nitrate to cauterize the bleeding.   Be very limited on how much silver nitrate us use to cauterize to avoid leakage onto the face.  In adults the key issue is BP control.  If the patient is very hypertensive, you will have limited success in stopping bleeding until BP is lowered.   Other key question is which side did bleeding start first.  Focus your control efforts on that side.   Lovell comment: Pain control and anxiolysis will also be key in controlling BP and bleeding.  Look for bleeding on septum, inferior turbinate, and then floor of nose.   If it is not in one of those 3 places then is likely lateral to middle turbinate or more posterior.   In the office, Dr. Marra will use an endoscope to see where posterior bleeds are coming from.  First line treatment for majority of nasal bleeding is still silver nitrate.   Dr. Marra will inject lido with epi 2ml in the anterior septum (junction of skin and septum) and floor of nose to stop brisk bleeding in the anterior nose especially if patient is hypertensive.  You can also inject at junction of nose to medial orbit angling toward forehead to cause arterial vasoconstriction.  In addition use topical vasoconstrictor in nose to temporarily halt bleeding so you can see better.   Preferred packs for Dr. Marra are Rhino-rocket and posterior balloon.  Problem with posterior balloon is that it retracts back into the nose and can cause alar necrosis or obstruct the airway.  If you use the posterior balloon, inflate the anterior balloon first so it doesn’t retract into the oral pharynx.   You have to use saline to inflate the posterior and anterior balloons.  Air insufflation does not provide enough pressure.   You can pack with Vaseline gauze around the anterior nose balloon to secure it better so it doesn’t retract into the nose and cause airway obstruction.   

Post tonsillectomy bleeding:  Most bleeds that end up in the ER are due to delayed bleeds (days 5-10).   Gargling with ice water will stop or decrease 95% of these bleeds.   Dehydration is a common factor that needs correction so IV fluids also indicated.  Dr. Marra wants most post-tonsillectomy patients admitted for OBS.   The basic ED treatment algorithm is have patient gargle with ice water until bleeding stops or until ENT can evaluate patient

Villano   Opioid Overdose

700% increase in opioid prescriptions between 1997 and 2007.

Opioids affect pain receptors, respiratory status, gut motility, pupillary constriction, and level of consciousness.

Opioid metabolism is very prolonged in an overdose situation,this includes fentanyl.  This is due to altered pharmokinetics in overdose.

 Methadone can cause QT prolongation.

No evidence of opioid use for acute pain  resulting in chronic dependency.

Toxic effects: respiratory depression which begins with decreased tidal volume, then decreased ventilator rate.

Case 1: Heroin overdose treated with narcan twice.  Pt was intubated for hypoxia due to acute lung injury.   Any opiate can cause this.  It is due to inspiration against closed glottis, hypoxic alveolar damage, and there is some controversy that narcan can also be a factor in this process.

Case2: Methadone OD treated with narcan.  Pt had cardiac arrest likely due to torsades from prolonged QT .   Methadone prolongs QT interval.

Case 3: Heroin OD treated with narcan. Pt found to have endocarditis while in hospital.   Gotta consider endocarditis or HIV in opioid abusers and OD patients.

Case 4: Chronic pain patient with altered mental status treated with narcan.  Pt responded and was sent back to jail.  Pt returned 3 days later with liver injury from APAP overdose.  Don’t forget that patients can be APAP or ASA toxic in addition to opioid toxic.

Narcan: can give subQ,  IV (best), intranasal, sublingual.  PO is poorly bioavailable.   Starting does is 0.05mg to 0.4 mg IV.  If no effect, you can increase to 2mg, then 4mg. Can actually go up to 10-15 mg if suspicion is very high for severe opioid OD.   In general, start low so as to avoid rapid severe opioid withdrawl.  Rapid severe withdrawl can be dangerous to staff and patient.

Case 5: Pt thought to have opioid overdose from wife’s meds and was admitted with opioid overdose. In hospital pt was found to have taken his daughter’s dilantin and was toxic from dilantin. Be sure to question and check for other ingestants/co-ingestants.

Case 6: pt with opioid overdose and treated with escalating does of narcan with good outcome.   In general, observe patient for 4-6 hours after narcan. If no recurrence of respiratory or mental status  depression, pt can go home.  If patient needs second dose of narcan or uses extended release preparation, admit the patient.  Put patients who OD’d on methadone, or fentanyl patches or extended release preparation in the ICU.  Narcan may be less effective in the elderly due to physiologic differences in seniors. 

 Pitfalls: Failure to escalate dosing of narcan.  Inadequate period of observation for recurrent respiratory depression.   Failure to look for acetaminophen toxicity.   Unecessary intubation. Incorrect diagnosis.

Coglan: You can inject narcan in sublingual area if no iv access.    Lovell comment: Re-emphasize the need to start with low dose narcan to avoid overly rapid and severe correction of opioid toxicity.  It puts your staff at risk when pt is rapidly reversed out of a coma . Chastain comment: Take a deep breath before intubating patient and give narcan first.  Intubation is not the first line treatment for opioid OD patients.   There was a generalized discussion about the fact that narcan should work rapidly in 4-5 minutes.  If effect is not rapid then probably opioid toxicity is not the cause of mental status change.

Felder/Urumov/Carlson  Dental Lab

 

Conference Notes 11-6-2012

Conference Notes 11-6-2012

Girzadas    Recruiting Season Prep

Lambert  Study Guide  Resuscitation

Discussion of management of witnessed VF arrest between Harwood and Lambert.  If you are by yourself managing the patient defibrillate first.  If you have someone else helping you, first  start CPR while another person gets the defibrillator.

 Procainamide infusion is stopped if you reach 17mg/kg, QRS widens by 50%, pt becomes hypotensive, or arrhythmia is terminated.

Most common common underlying cause of CHF is coronary artery disease.

Cuffed ET tubes in pediatric patients are recommended to eliminate oropharyngeal contamination into the airway.

Co2 detectors on ET tubes can have false negative results if pt has no circulation.  Meaning the ET tube can be in the trachea but patient with no circulation is not expiring any CO2

There was a discussion of the appropriate tidal volume for ventilated patients between Lambert and Barounis.   Smaller tidal volumes are becoming more recommended 6-8ml/kg for all ventilated patients is now considered lung protective.

There was a discussion of whether or not to sedate patients prior to giving adenosine.  No general agreement about the necessity of this.  Most did agree that if there is time and adequate help, giving sedation is the kind thing to do to limit pt discomfort with adenosine.

Most common adverse effect of IV amiodarone is hypotension.

IV diltiazem is contraindicated in wide complex tachycardia.

In a choking patient, if they can speak or cough do not intervene.  Let them try to clear their throat spontaneously.

Eastvold        Acute Decompensated Heart Failure

There are few guidelines regarding acute decompensated CHF.  This is because most decompensated CHF patients in the ER are suffering from diastolic dysfunction and most CHF research is on patients with systolic dysfunction.

Pearl #1 Chf is not gout.  It is not a one treatment/one approach illness.

Up to 50% of patients arriving to the ED with pulmonary edema are not fluid overloaded.  However, afterload mismatch/vascular failure  can cause body fluid to be shifted to the lungs even if the patient is not total body fluid overloaded.

In a patient with acute chf who is hypertensive, they are less likely to have systolic dysfunction.

Pearl #2 Pts with CHF and HTN suggest vascular failure and fluid predominantly shunted to lungs with diastolic dysfunction.

5 reasons the patient in front of you doesn’t have  chf: no hx of chf, no DOE, no rales, no cardiomegaly, nl ekg, BNP<100.  The more of these that are present the less likely it is to be CHF.

BNP may be falsely low due to flash pulmonary edema (it takes time to increase the bnp) or acute mitral insufficiency.

Vascular pedical width (width just below the aortic knob >71mm) and cardiomegaly on CXR increase chance of chf.   Using the cardiac or abdominal probe, Ultrasound findings of at least  4 lung rockets/b-lines/headlights/searchlights in the fog are indicative for alveolar/interstitial fluid. Put the probe on the anterior chest bilat.    This ultrasound finding can be due to pulmonary edema, pulmonary fibrosis, or Aids lung and a few  others.

 PT with CHF and a Bun>43, SBP<115, Cr>2.75=22% mortality in hospital.   Elevated troponin and hyponatremia are another two markers for high risk CHF.

To categorize CHF patients in the ED look at the BP

CHF with HTN (>140 systolic): Rapid onset.  Fluid is in the wrong place (vascular failure).  Bipap and nitrates are the work horses for this category.  +/- Lasix 20-30 min after treatment started.    For the severely dyspneic patient with very high BP, you can load the pt with NTG by giving multiple suplingual sprays.     

Normotensive: Gradual onset.  Chronic CHF patients.  Give NTP or IV NTG and lasix. These patients can have a high mortality though.

Hypotensive: Cardiogenic shock. 

CHF with ACS: This does not mean isolated troponin leak.  Pt’s should have chest pain, ekg changes or pattern of troponin changes.

Isolated RV failure: nt discussed in this lecture

Morphine: Increases mortality and rate of intubation and ICU admission.

Acute CHF patients only have 20% of normal renal flow.  This risks renal injury from lasix if you over diurese.  It also may not be much help in patients with vascular mismatch CHF.  If you use lasix, start with low dose.  If the patient is on lasix at home, match their usual po dose with iv Lasix.  Josh feels EMS should not be giving lasix because it is often misused.

NTG is the drug of choice for decompensated heart failure.

IV ACE-I is usually not needed if you are using NTG aggressively.

BiPap has been shown decrease mortality.  Use it for patients who arehypoxic and/or dyspneic.

Harwood comment: Give 4 sprays of NTG pretty much out of the gate to the severely hypertensive patient.   Intubating a acutely decompensated CHF patient  should be considered a failure of EM management.

Tomasello     ST Depression on EKG

ST depression in AVL suggests inferior wall AMI.  Get serial ekg’s and watch for inferior ST elevation.

High lateral AMI’s are usually due to circumflex occlusions and mostly don’t have much ST elevation because the high lateral region has low voltage generally.  Look for minimal st elevation in high lateral leads and inferior st depression.

Think posterior MI with tall anterior R wave, horizontal st depression anteriorly, upright t waves anteriorly. Posterior MI’s  affect  the posterior or far lateral aspect of the left ventricle.

 Discussion about DeWinter wave vs. Wellen’s.  Both are related to LAD occlusion and are found anteriorly, but Wellen’s is usually seen when pt is pain free.

With an EKG with diffuse ST segment depressions think either three vessel disease or left main (left coronary prior to bifurcation of LAD and circumflex)occlusion.   You need st depression in 6 leads or more with st elevation in AVR  to call left main.  Most need CABG so don’t give plavix.  

 

Ryan   Med Student Review

 

 

 

Conference Notes 10-30-2012

Conference Notes 10-30-2012

Barounis  Peds Joint Conference Bronchiolitis

Case 1: 7wk child with bronchiolitis HR=180, T=39,  P/O=97%

Is albuterol indicated:  Suctioning is critical.   Nasal occlusion may make it that you don’t hear wheezing.  Dr. Horowitz  from ICU said a trial of one albuterol or one racemic epi is reasonable.   Dr. Akhter also felt a trial of albuterol is reasonable. If there is a response you can continue with nebs.   Dr. Akhter said if child has risk factors for asthma like parental history  or food allergies then he is more likely to benefit from  bronchodilators.  Dr. Roy said this decision making tool is more useful in an older kid.   Dr. Roy discussed one hospital system that had a “suction shack” aside from the ED that kids with bronchiolitis would go to multiple times per day for suctioning.  He continued that in a child with distress, it is hard not to use bronchodilators.   Barounis talked about the Cochrane Review that found that there was no benefit from bronchodilators in bronchiolitis.  Dr. Roy said that if there is no response to initial few nebs then stop.  Dr. Akhter said though that it can be hard to guage patient resonse to bronchodilators.

Hypertonic Saline:  Dr. Bill Schroeder says no evidence for effectiveness.  Dr. Horowitz  agreed.  Dr. Akhter said 3% saline nebs are for bronchiolitis, 7% saline nebs for CF.   Barounis showed the Cochrane review that demonstrated  3% saline shortens hospital stay by about a day.   Dr. Akhter said that the Cochrane Review may have suffered from author bias.  Akhter “I have been less than impressed with the effectiveness of 3% saline.  But it is a benign therapy that is safe to use.”

Racemic Epi: Bill Schroeder uses with the sicker kids as a trial of therapy.  Occaisionally it helps.  Dr. Horowitz uses in kids with nasal plugging predominantly  to gain the alpha effect  to vasoconstrict the nose and upper airways.  Dr. Akhter uses it empirically as a trial and if it helps you can continue.  Dr. Roy said  this whole discussion is frustrating because there is no treatment that has clear data showing improvement.   

Steroids:  Dr. Bill Schroeder does not routinely use steroids unless there is a family hx of asthma and child is improving from bronchodilators.   Horowitz does not use steroids in a child this age.  He will use steroids in an older child who may have asthma.  Dr. Roy said no unless there is risk for asthma.   Barounis- how about a 12 month old kid who responds to bronchodilators?  Akhter said Cochrane review demonstrated no benefit in bronchiolitis.  However, if you think it is asthma give steroids.   Lovell comment: If steroids are given for bronchiolitis at first visit, then when they come back 2 months later with a respiratory illness steroids are often given again for probably no good reason.  Akhter responded that if kids are returning to ED multiple times then they likely do have asthma an may benefit from steroids.

Racemic Epi and Steroids:  NEJM Article showed some benefit.   Dr. Roy questioned the validity of the study and suspects bias in the study group looking at racemic epi for bronchiolitis.  He did not want to give a lot of steroids to possibly prevent some admission.

Dr. Collins question,  3% saline nebs for outpatients?  Akhter yes it is safe to use.    Harwood question: How many children have died at Hope from bronchiolitis?   Horowitz: usually they do fine but some end up on ventilators.   Congenital heart disease kids can have problems.   Dr. Roy  said that he can’t recall any kids dying from isolated bronchiolitis.    Dr. Harwood said if kids can feed and their O2 sats are 92% or higher they can be discharged after suctioning.   All these kids survive.   Very young and those with congenital heart disease are higher risk.    Another audience membr  comment is that Medicaid will not cover home suction machines this winter.   We need to get Nosefrieda devices in the ED.

CXR:  Dr. Akhter said  the CXR has only about a 1% diagnostic yield in kids with likely bronchiolitis.   Bill Schroeder doesn’t routinely get CXR for bronchiolitis.   Dr. Horowitz said babies with bronchiolitis can specifically get RUL atelectasis (considered fairly specific cxr finding).  He also brought up to not forget about UTI in febrile kids with bronchiolitis.

Should we test for RSV: Sirosek-NO.  It has no utility during bronchiolitis season.  Dr. Butterly said test may have some risk stratification utility. Dr. Collins said it has no utility when we have single patient rooms.   Dr. Roy and Dr. Collins said we should be isolating by symptoms not by RSV testing.

Summary by Dr. Barounis: Suction is mainstay of treatment. Give trial of albuterol, can try 3% saline nebs,  avoid CXR, avoid  Steroids, avoid testing.   Racemic epi can be tried in the very nasal congested kids or sicker kids. 

Patel/Katiyar  Oral Boards

Case 1 Iron poisoning: Treat with IV fluids, intubation, give deferoxamine, consult poison control.  Most common fatal pediatric overdose.   Look for a high anion gap metabolic acidosis  (It is one of the I’s in MUDPILES).   Iron is caustic to gi tract/produces free radicals/disrupts ox-phos in the mitochondria.    Check  two Iron levels separated by 2 hours.  If sustained released or enteric coating get levels several hours later as well. Treat with deferoxamine.  This will give you a vin rose color of urine as complexed iron is excreted. 

Case2 Herpes keratitis: Need to consult ophthalmologist.  Educate patient about potential blindness.    Corneal findings can be an ulcer or dendritic pattern. 

Case3 Imperforate Hymen:  Hymen obstructs menstrual outflow causing hematometria (blood  in uterus) or hematocolpos (blood in the vagina).     Presentation is adolescent with abdominal pain and amenorrhea.  Can be a monthly cycle.  Hymen may be bulging in exam.   Treatment is surgical incision of hymen.

Lovell comment:  With Tox cases, your goal is to figure out the toxin.  Get info from parents/family/ems/ pmd or drug store.  Always check for visual acuity on all eye cases. Visual acuity is the vital sign of the eye.

Coghlan comment: Be sure to circle back on the tests you order like accucheck.  Check blood sugar prior to intubating a patient with altered mental status.

Anneken     Electrolytes/Acid Base

Moderate dehydration should be treated with oral rehydration.  Solution should have sodium/glucose.  Not too much glucose to cause diarrhea

Kids with diarrhea may return to a regular diet as soon as tolerated. It has been shown to shorten th course of diarrhea.

Calcium gluconate is the treatment of choice for wide QRS due to hyperkalemia.   It stabilizes cardiac membranes but does not decrease k+ . It is the fastest acting treatment.    Insulin/glucose is the best treatment to decrease serum K level.  

Severe Hyponatremia (<120) associated with severe symptoms (coma/seizures/focal neuro findings) treat with hypertonic saline.   Also use for acute drops in sodium (marathon runners/ecstasy use/polydipsia).   Down side to hypertonic saline use is overly rapid correction resulting in Osmotic demyelination syndrome.       Harwood comment: He is not using hypertonic saline unless patient is in status epilepticus or not protecting airway.  It has been recently been found that ODS can be stopped by giving water.

Sorry I missed the rest of this lecture.

Barounis   Electrolyte Emergencies

5 causes of hyperkalemia:  renal failure, meds (example pt is on an ace-i  and you give them an NSAID),  cell death like tumor lysis/ischemic gut/rhabdomyolysis.

Ekg changes due to hyperkalemia: peaked t waves, prolonged QRS, loss of p waves, bradycardia , sine wave, heart block.  Elise comment:  If ekg is real wide and ugly that is a classic sign of a metabolic problem/diagnosis.   There is no linear stepwise change in ekg correlating with K level.  However if there are EKG changes, the K level is likely over 6.5.

If giving insulin for hyperkalemia, give 1 amp of glucose for every 5 units of insulin.  Example is 2 amps of glucose for 10u of insulin.

IV bicarb works well for hyperkalemia only if patient is acidotic.

If giving calcium,  calcium chloride has 3 times more calcium than gluconate.  If pt is unstable give calcium chloride IV.   Calcium gluconate is broken down in the liver and takes more time (20-30 min) to be effective.  Ca Chloride is very sclerosing and otentially tissue damaging.  There was a discussion of administration of CaChloride and Cagluconate.   It was generally agreed that Cagluconate should be used in patients who don’t have immediately life threatening hyperkalemia.   CaChloride should be used in patients with immediately life threatening hyperkalemia.

Down-Up pattern of ST segments is a sign of hypokalemia in addition to U waves and prolonged QT.  Hypokalemia can be a cause of death from V-fib.  Hypokalemia=Hypomagnesiemia.

If a patient has a K of 3.0, they are whole body depleted of potassium by 300meq.   You can’t replete 300meq in the ED.  The best you can do is 10meq per hour via a peripheral IV.   Max rate of repletion is 20meq/hour via central line.  What you can do is give patients some repletion in the ED and figure out why they are losing potassium at home (diuretics).   Pt’s should also eat 2 bananas/day or drink OJ every day. 

Ecstasy use can result in hyponatremic seizures.  More common in women.    

If a baseline normal patient has hyponatremia less than 120 and is seizing  (acute severe hyponatremia), give hypertonic saline 100ml over 10 minutes.  Can push or give IVDrip or IV Pump.  Alternative is to give 1 amp of sodium bicarb (has 50meq of sodium just like 100ml of hypertonic sodium).    If patient has mild symptoms (weakness, fatigue), just do nothing except consulting nephrology.   For all hypontremia patients, never correct more than 10 in a day.

Hypercalcemia: 90% are caused by parathyroid adenoma or other cancer.   Treat with 200ml of saline hydration per hour.   2nd-3rd-4th line  therapy is diuretics and bisphosphonates and calcitonin.

CAT MUDPILES

Cyanide, CO, APAP, Toluene, Mthanol, Uremia, DKA, Paradehyde, Porpylene glycol, iron, isoniazid, lactic acidosis, ethanol, ethylene glycol, salicylate.

Chastain  Electrolyte Abnormality Cases

Seizing patient due to severe hyponatremia: Treat with 100ml of hypertonic saline or 1 amp of sodium bicarb over 10 min.  Can repeat if needed.

 

Conference Notes 10-23-2012

Conference Notes 10-23-2012

Lovell  Study Guide Pulmonary

Massive hemoptysis: 600ml of blood in 24 hours.   For an ED perspective, hemoptysis that impairs oxygenation/ventilation.   Stabilize with likely bleeding side down.  Get them to CT and consult IR for bronchial artery embolization.

Tracheo-Innominate Artery Fistula:  Usually in the first several weeks after placement of trach (85% occur in first month).  Can have herald bleed followed by exsanguination or severe bleeding impairing ventilation.  Treatment is putting your finger in trach hole and applying anterior pressure against the sternum.  Other option is to over- inflate trach cuff.

Hermmann question followed by Harwood, Lovell comment: You have to work up 100% of patients with Trach tube and bright red bleeding from trach with CTA or bronch.  More likely in patients who had trach placed in the last month.

 Platypnea: SOB when sitting up. It is the opposite of orthopnea.   Elise’s factoid.

Aspiration pneumonia: CXR findings develop in dependent portion of lung.  Aspiration initially is a chemical pneumonitis followed by pneumonia.  No prophylactic antibiotics and no steroids.  There is controversy about NH patients whether to start antibiotics with early signs of aspiration.

FDA approved method of treating hiccups is Chlorpromazine (Thorazine).   Harwood method is having patient suck water through a straw while they holding their fingers in their ears.    Alternative home method is dry granulated sugar in the back of the throat.

Before you put a chest tube in a COPD’r is make sure what you think is a pneumo is not really a big bleb.  If they are SOB treat with nebs and steroids, don’t put in a chest tube. 

Deep sulcus sign is highly suspicious for a pneumothorax.  The costophrenic angle dips much further inferiorly on the affected side than on the unaffected side.

Catch phrase for Legionella pneumonia:  dry cough with diarrhea.

Catch phrase for Strep pneumonia: rust colored sputum

Catch phrase for Staph aureus: post-influenza pneumonia

Catch phrase for Klebsiella: alcoholic or NH pt, abscess

Catch phrase for Mycoplasma: upper airway and lower airway symptoms, bullous myringitis

T B: Pott’s disease is TB to the spine.  Scrofula is large lymphadenopathy in the neck due to TB. Gohn complex (pulmonary scarring with hilar adenopathy)is classic CXR finding of latent primary TB.  These patients  are treated with 9 months of INH similar to newly positive ppd .  Reactivation TB is active TB and need 4 drug treamtment.   Miliary TB is diffuse lung findings in an immunocompromised patient means TB out of control.

Spontaneous pneumothorax in young stable patient: Catheter aspiration or Heimlich valve techniques are superior to placing a standard chest tube.

Harwood comment: Can you extrapolate this data to iatrogenic pneumothorax? Elise  response: yes.

Elise said she would observe any patient treated with catheter aspiration for 6 hours in the ED.

Villano/E Kulstad  Oral Boards

Case #1 Acute MI: ASA/O2/screen for contraindications/give thrombolytic.  Start IV heparin.

Case#2 Acute Angle Closure Glaucoma: Identify diagnosis, start treatment promptly, consult ophtho.  Look for iris bowing forward on slit lamp exam.  You can see the light beam with curve over iris.  Treat with timolol/apraclonidine/pilocarpine/prednisolone/acetazolamide.  If IOP still over 40 give mannitol.  Treat pain and nausea.

Case#3 Digital hair tourniquet:  Identify affected finger and remove tourniquet.  The third toe and third finger are the most common digits involved. There is an association older clothing and mittens.  Bill Schroeder comment: use intranasal fentanyl prior to digital block.   Removal with fine scissors/scalpel and forceps.   Sarah Herron comment: Nair removal takes too long.  Bill Schroeder comment: you don’t always know that the tourniquet is hair so Nair may not be effective if not hair.  It could be synthetic/thread.

Schroeder  Peds Resuscitaiton

ET tube size: (age/4 +4) for uncuffed.   (Age/4) +3 for cuffed tube.  Easiest way to calculate is a Broslow tape.

 EPI dose is 0.01mg/kg or 0.1ml/kg of the 1:10,000 for PEA.

Ketamine sedation is 1-2mg/kg IV.

Treatment for TCA OD is sodium bicarb.  Dose is 1meq/kg.   Amp of 8.4% bicarb has 1meq/ml or 50meq in an amp.

Lower limits of systolic BP is 70mm hg + (age x2).  Under one month is 60mm hg.  90mm hg for ages 10 and up.

Goal of fluid boluses in the first hour for the critically ill can be up to 60ml/kg.  This is done by repeated 20ml/kg boluses as needed.

IM versed 0.01mg/KG is the fastest way to get anti-seizure meds into a child with no iv access until you get an IO or IV line.

Atropine for brady arrest dose is 0.02 mg /kg.  Minimal dose is 0.1mg and max dose is 1mg.

The  first line pressor for peds patient with volume refractory shock is dopamine.  The reason is that kids generally have an already high SVR in shock.

Give pyridoxine for refractory seizures in kids.   Can be due to B6 deficiency in a neonate or infant.  Also can be used for INH related seizures.

D50 in an infant can cause cerebral edema due to hyperosmolarity.  It can also cause tissue damage in the extremity if it extravasates.  It can also lead to hyperglycemia followed by rebound hypoglycemia.

Succinylcholine may cause bradycardia in young children and infants.  Can use with atropine. Can cause hyperkalemia in kids with muscular disorders like muscular dystrophy.  Rocuronium is an alternative but has slower onset of action than succinylcholine.  Bill personally uses rocuronium on all infants he intubates.

Ductal dependent lesions causing shock in infants can be treated with prostaglandin IV.

Bill Schroeder comment: In a critical case, don’t be calculating doses, use the broslow tape or a pediatric dosing app on your phone. 

Klinker/Wolfe Inhaled Medications in Asthma

Atrovent MDI’s  have peanut oil and should be avoided in kids with peanut allergy.

Conversion: Albuterol 5mg= 4 puffs Q30 minutes.  10mg=8puffs Q 30min.    15mg=8puffs Q20 min.   20mg=11puffs Q20 minutes.

Home treatment should be 4puffs QID to Q 4hours.  If child is having difficulty breathing advise parents to continue to give puffs on the way to the ED.   Bill Schroeder said his rule of thumb is if child is requiring 8 puffs at a time or needs mdi tx more frequently than q 4 hours return to ER.

 Bronchiolitis:  Kids under 4 months are obligate nose breathers.  If their nose gets stuffed up, they have a lot of trouble breathing.   Don’t routinely do a CXR.  Treat with suctioning, maintain hydration, O2 to keep sat over 90%.  When suctioning with bulb syringe tell parents to stick it into the nose and aim at the toes.   NoseFrida is a suction tool that parents can suck on to remove mucous from the nose.  On the NoseFrida website you can find a local store that has this device.  Parent who have used this rave about it.  Good suctioning can improve upper airway movement and decrease wheezing in lower airways. Antibiotics, CPT and steroids are not recommended.  For sicker bronchiolitic patients you can try albuterol or hypertonic nebs.

Conference Notes 10-9-2012

Conference Notes  10-9-2012

McDermott/C. Kulstad  Oral Boards

Sorry I missed this Oral Boards Triple Cases but the highlights per Dr. Kulstad were:

Case 1: spinal shock fro cspine injury:  treat with iv fluids, pressors if needed.  Board Question Alert! Pt’s may be paradoxically bradycardic with hypotension.  Neurogenic shock refers to hypotension, usually with bradycardia, attributed to interruption of autonomic pathways in the spinal cord causing decreased vascular resistance. Patients with TSCI may also suffer from hemodynamic shock related to blood loss and other complications. An adequate blood pressure is believed to be critical in maintaining adequate perfusion to the injured spinal cord and thereby limiting secondary ischemic injury. Albeit with little empiric supporting data, guidelines currently recommend maintaining mean arterial pressures of at least 85 to 90 mmHg, using intravenous fluids, transfusion, and pharmacologic vasopressors as needed

Case 2: Heat stroke: Rapid cooling.  Altered mental status  separates heat stroke from heat exhaustion.

Case 3: Stingray injury:  Hot water treatment and don’t close wound.  Xray to make sure no fb

Kessen   Hand Trauma (Sorry missed a lot of this lecture)

Jersey finger is due to rupture of the flexor tendon. Pt can’t flex finger.  Called Jersey finger because football players would get this when tackling someone by grabbing their jersey.

For amputations: get all the pieces and x-ray all the pieces.    Digit survival is 12 hours when warm, 24 hours if cooled.    Major replant survival is 6 hours warm,  12 hours cold.     Keep amputated digit cool by wrapping in saline soaked gauze, place in a plastic bag and put the plastic bag on ice.  We have a cool to keep on patients cart.  

Harwood comment: MRI can be used to identify FB and many specialists have access to MRI in their offices.   Using a tourniquet to get a bloodless field will be less painful for a patient if you keep the cuff pressure only 20 mm hg above the patient’s systolic BP.  Current standard of care is that EP’s don’t do tendon repairs in most areas of the US.   Hand specialists and ortho specialists will take almost all tendon injuries.  

 

 

 

Discussion of Regional anesthesia for the hand.   See Diagrams Below

 

 Flexor tendon approach

 

 Web space approach

 

 

Levato   UTI treatment  (I missed a lot of this lecture also)

Uncomlicated UTI’s use macrobid for 5 days or keflex for 7 days.  Bactrim has too much resistance to be considered reliable.     Cipro should be used only if other options not possible because cipro use has complications of c-diff/neuro effects/tendonopathy/interactions with Coumadin.  Use cipro for only 3 days for cystitis.

Collander  Unstable C-spine Injuries

Intubate for Cspine fractes C5 or higher.

Rectal tone presence identifies incomplete cspine injuries.

Nexus criteria are 99.6 % sensitive for clinically significant Cspine injuries.

The NLC decision instrument stipulates that radiography is not necessary if patients satisfy ALL five of the following low-risk criteria:

  • §  Absence of posterior midline cervical tenderness
  • §  Normal level of alertness
  • §  No evidence of intoxication
  • §  No abnormal neurologic findings
  • §  No painful distracting injuries

Insignificant injuries were defined as those that would not lead to any consequences if left undiagnosed. The NEXUS investigators evaluated 34,069 blunt trauma patients who underwent radiography of the cervical spine comprised of either a 3-view cervical spine x-ray or a cervical spine computed tomography (CT) scan. Of these patients, 818 (2.4 percent) had sustained a cervical spinal column injury. Sensitivity, specificity, and negative predictive value (NPV) of the NLC were found to be 99.6 percent (95% CI 98.6-100), 12.9 percent (95% CI 12.8-13.0), and 99.9 percent (95% CI 99.8-100), respectively

 

Canadian Cspine rule is 100% sensitive for clinically significant spinal injury.

The CCR involves the following steps:

  • §  Condition One: Perform radiography in patients with any of the following:
    • ·         Age 65 years or older
    • ·         Dangerous mechanism of injury: fall from 1 m (3 ft) or five stairs; axial load to the head, such as diving accident; motor vehicle crash at high speed (>100 km/hour [>62 mph]); motorized recreational vehicle accident; ejection from a vehicle; bicycle collision with an immovable object, such as tree or parked car
    • ·         Paresthesias in the extremities
  • § 
    • ·         Simple rear end motor vehicle accident; excludes: pushed into oncoming traffic; hit by bus or large truck; rollover; hit by high speed (>100 km/hour [>62 mph]) vehicle
    • ·         Sitting position in emergency department
    • ·         Ambulatory at any time
    • ·         Delayed onset of neck pain
    • ·         Absence of midline cervical spine tenderness

Patients who do not exhibit any of the low-risk factors listed here are NOT suitable for range of motion testing and must be assessed with radiographs.

If a patient does exhibit any of the low-risk factors, perform range of motion testing, as described in Condition Three below.

  • §  not

In the derivation study, the CCR demonstrated a sensitivity of 100 percent and a specificity of 42.5 percent for identifying clinically important cervical spine injuries

 Flexion teardrop fracture: Anteroinferior portion of vertebral body is fractures off.  Can have associatated anterior cord syndrome.   May have widening of spinous process spaces.

Wedge Compression fracture:  Posterior ligament disruption may be associated.  Considered unstable if >25% compression of the anterior border of the vertebral body or widening of the spinous processes.

Extension teardrop fracture: Anteroinferior portion of vertebral body is avulsed.  Fragment is usually taller than wide.

Hangman’s fracture: Fracture of both pedicles of C2. C2 displaces anteriorly.  Usually see in car and diving accidents.  Patients can be neurologically intact because there is a wide canal at that level.

C1 Jefferson Burst Fracture: Due to an axial load.  C1 is laterally displaced on C2.   If sum of total displacement of lateral masses from body of c2  is greater than 7mm that is the criteria.

 

Occipital-atlantal Dissociation: figure

 

The Powers ratio is commonly used to assess for atlanto-occipital dislocation (figure 9). It is defined by the ratio of BC:OA, where BC is the distance between the basion and the midpoint of the posterior laminar line of C1, and OA is the distance between the midpoint of the posterior margin of the foramen magnum (opisthion) and the midpoint of the posterior surface of the anterior arch of C1 [17]. A ratio greater than one suggests anterior subluxation.

Another radiologic finding suggestive of an atlanto-occipital dislocation is disruption of the “basilar line of Wackenheim,” a line drawn from the posterior surface of the clivus to the odontoid tip [18,19]. Normally, the inferior extension of this line should just touch the posterior aspect of the tip of the odontoid. If the line runs anterior or posterior to the odontoid tip, this suggests an atlanto-occipital dislocation.

Carlson  Salicylate Toxicity

1960’s there was concern for ASA causing Reyes syndrome and people were told not to have asa at home.  Toxic ASA exposures decreased for a few decades because people didn’t keep ASA at home as much.  Since the 1990’s ASA use has again resurged due to it’s value for cardiac disease.

Board Question Alert! Oil of wintergreen has a very high concentrate of methylsalicylate.   7grams of ASA in a teaspoon!

Enteric coating of ASA prolongs absorption to 4-6 hours and asorption is less predictable.

ASA inhibits cycloxygenase to block prostaglandin synthesis.  Toxic levels stimulate respiratory center (respiratory alkalosis), stimulates vomiting center, increased capillary permeability (pulmonary edema) and uncouples oxidative phosphorylation (metabolic acidosis, fever).  Pts will develop ketosis and hypokalemia in addition to metabolic acidosis and respiratory alkalosis.  Toxic patients also will have tinnitus.

More severe toxicity will cause agitation, dehydration, acid/base disturbances, pulmonary edema.

A death from ASA is a CNS death.   ASA is a brain poison.

ASA poisoning gets missed  because it looks like sepsis or alteredmental status or chf.

Toxic dose is >150mg/kg.   Serious toxicity can be approximated by 1 (325mg) tab per kg.  therapeutic level of salicylate is 3-6mg/dl,  toxic level is >30mg/dl.   Levels correlate poorly with toxicity.  Done nomogram is no longer used because it is inaccurate.   Don’t use the Done nomogram. 

Board Question Alert!    If you need to intubate a patient with severe ASA toxicity or any patient who is markedly tachypneic, be sure to set your ventilation parameters  to maintain the patient’s minute ventilation so they don’t become more acidotic.

Management: Activated charcoal,  additional dose 2 hours later of activated charcoal,  alkalinize blood and urine (target urine ph is 7.5-8),  need to keep potassium in normal range or you will not be able to effectively alkalinize the urine.  You will usually need to hang a lot of potassium.   Hemodialysis is indicated for severe overdoses. (acute level>100, chronic level>60, pulmonary edema, renal failure, pulmonary edema, rapidly rising levels, altered mental status and academia.

,

Mistry  FirstNET EMR

Tech support continues thru 10-17.  Make sure you work some shifts/see some patients while tech support is her on site.

Chintan went through multiple optimizations of First Net.

Conference 10-2-2012

Conference Notes 10-2-2012

Gottesman/Anderson   Oral Boards

Case 1: CO poisoning

Case2: AKA:  Treat with IV fluids and glucose

Case3: Morbidly Obese Patient with respiratory failure:  

Harwood comment: The lesson of these 3 cases is getting the ABG. It will help you solve all 3 cases.  VBG can be used frequently in place of ABG.  If you want a CO level on the VBG, make sure to tell the respiratory therapist. They may not run the CO.    The caloric content of a bag of D5 is 200 calories, so you may need to give D10 or food or Amps of glucose to correct marked hypoglycemia.

Girzadas comment: For the SuperObese patient be sure to use RAMP positioning and call for back up from anesthesia or other EM physician

Kulstad Study Guide CV Disease

Aortic insufficiency murmur is heard in 32% of patients with aortic dissection.

Best work up for iliac dvt in a pregnant patient is MRI.

Work up for ischemic limb is Vascular consult and CT angio of limb.  Heparin is usually indicated.  Definitive therapy is thrombectomy and embolectomy.  Harwood comment: Just give heparin and consult vascular surgery.  Ct angio may be a time waster.

Signs of Aortic dissection: wide mediastinum, tracheal deviation, and aortic shadow beyond calcified wall.   Harwood comment: there is a difference between traumatically torn aorta and aortic dissection.  Xray findings are not all common to both.

Phlegmasia cerulea dolens: Severe ileo femoral dvt with venous engorgement.  Can lead to compartment syndrome and gangrene.   Treat with heparin and IR thrombolytics.   Phlegmasia alba dolens is called the milk leg.  Much less common and is a dvt resulting in decreased arterial perfusion.

High risk patients with concern for dvt who have a negative U/S, they need f/u U/s in 7 days.  2 negative U/S makes risk of PE or DVT less than 1% in 3 monts

Wells criteria and modified Wells criteria: clinical assessment for pulmonary embolism

Clinical symptoms of DVT (leg swelling, pain with palpation)

3.0

Other diagnosis less likely than pulmonary embolism

3.0

Heart rate >100

1.5

Immobilization (≥3 days) or surgery in the previous four weeks

1.5

Previous DVT/PE

1.5

Hemoptysis

1.0

Malignancy

1.0

Probability

Score

Traditional clinical probability assessment (Wells criteria)

High

>6.0

Moderate

2.0 to 6.0

Low

<2.0

Simplified clinical probability assessment (Modified Wells criteria)

PE likely

>4.0

PE unlikely

≤4.0

Data from van Belle, A, et al. JAMA 2006; 295:172.

 

Most common extremity aneurysm is  popliteal .  Often bilateral and rarely rupture.

Treatment of aortic dissection: reduce shear force with esmolol and drop MAP to 60.  Can add nitroprusside or other agent if needed to get map to 60.  Start though with esmolol.    Labetalol would be another option.

Be very cautious managing asymptomatic htn.   There is a risk of causing stroke with rapid lowering of BP.  Restart their medications.  If they are untreated you can start a low dose diuretic.

Thrombotic cause is more common than embolic cause of limb ischemia.   This is due to good anticoagulation management of patients with Afib and valve replacements.

There is no distinct number that identifies a hypertensive emergency.   Emergency is defined by end organ damage.   Harwood comment: pre-ecclampsia  is a hypertensive emergency with a relatively low bp cutoff.  Usually 140/90.

The following eight factors constitute the PE rule-out criteria

  • §  Age less than 50 years
  • §  Heart rate less than 100 bpm
  • §  Oxyhemoglobin saturation ≥95 percent
  • §  No hemoptysis
  • §  No estrogen use
  • §  No prior DVT or PE
  • §  No unilateral leg swelling
  • §  No surgery or trauma requiring hospitalization within the past four weeks

Coghlan comment: Why not include cancer in the PERC rules.  Barounis response that when Jeff Kline discussed on previous podcast he said cancer did not change the probability in his study.  

Harwood and Barounis felt that if a cancer patient has neg perc/neg dimer/neg trop then they likely don’t have a PE.   Elise and Barounis disagreed on whether CT would be indicated in this situation.  There was some heated discussion between Harwood/Elise/Christine/Barounis on this topic.  There was not consensus on whether a CT was absolutely necessary in the cancer patient who has a neg perc/neg dimer/neg trop.

Best treatment for  patient with asymptomatic htn who is not on meds currently:   HCTZ, Lisinopril .  Pharmacy student comment: Lisinopril may be less effective in African American patients.  Harwood comment: Chlorthaladone is a thiazide diuretic that is more potent than HCTZ.  Consensus was that you don’t need to start potassium therapy with low dose HCTZ or Chlorthaladone.  Hypokalemia is not a big problem with HCTZ 25mg or less.

Maslar  Dive Medicine

Humans can’t breathe under water through a long snorkel tube because there is water pressure pressing on our chest and increasing the air pressure in our bodies.  Our diaphragms cannot overcome this pressure.

Dybarism: most common source of diving problems. Ear squeeze is usually a problem of descent. Ear pain can develop.  TM can rupture.   Valsalva is treatment for ear squeeze but If you overdo it you can cause round window rupture resulting in hearing loss/vertigo/tinnitus.   Sinus pain is usually a problem of ascent resulting in sinus pain.  Pulmonary barotraumas can also occur on ascent in a diver breathing pressurized air.  The diver  needs to exhale as you ascend or the expanding air can cause alveolar rupture.  Patients can have pneumomediastinum.   Worst case scenario of dysbarism is air embolism.  Arterial gas embolism will occur almost immediately upon surfacing.  Of all dysbarism injuries, only the air embolism requires hyperbaric treatment.

Diver descending: ear squeeze.   Diver ascending:sinus pain, pulmonary barotraumas, arterial gas embolism

Decompression sickness (Bends): Usually involves nitrogen which is most prevalent atmospheric gas and is inert.  Joe used the can of coke metaphor to describe decompression sickness.  If you open a can of coke real fast you get a lot of bubbles. If a diver surfaces too fast relative to the time they were underwater you get bubbles in the blood/tissues/joint.  Interestingly, we don’t really know where bubbles come from or how they hurt us.   Gasses coming out of solution with decompression sickness usually affect the spine rather than the brain. Acute stroke symptoms should point more to arterial gas embolism than decompression sickness. Treatment is hyperbaric oxygen to push bubbles back into solution.

Who needs hyperbaric recompression tx: decompression sickness, arterial gas embolism, CO toxicity

Christine comment: If you have to treat a patient with a diving related malady and have questions or need guidance you can call the Diver’s Alert Network (DAN).

Lovell    Targeted Temperature Management Post-Cardiac Arrest

Post cardiac arrest syndrome: precipitating disorder, tissue ischemia,

Therapuetic hypothermia: mechanism of action is to slow down brain/heart/overall metabolism and slowing the inflammatory cascades that are negatively impacting brain.

Ice packs have been shown to effectively cool patients.  So low tech cooling means have been shown to be just as effective as the hi tech options.  We have cold IV saline in the ER to use to start cooling patients early.

Hypothermia therapy either results in patients with good neurologic outcomes or they die.  Hypothermia treatment does not result in more patients with a persistent vegetative state.

Number needed to treat for therapeutic hypothermia: good neuro outcome=6,  lower mortality=7.  These are great numbers!

 AHA guidelines: Class 1 recommendation for comatose patients with ROSC after V-Fib arrest. Should also consider with patients resuscitated from other types of arrests.

Complications to be expected: infection and coagulopathy, bradycardia, electrolyte abnormalities, 5-20% rate of seizures, labile BP, hyperglycemia, avoid hyperthermia with re-warming.  Keep patients below 37.5C.

Can use therapeutic hypothermia even if prolonged resuscitation and/or unwitnessed arrest or prolonged down time prior to resuscitation or cancer.  Elise made a strong point that in all decisions to initiate hypothermia treatment to consider their pre-arrest health status and pre-arrest prognosis. Can even use therapeutic hypothermia when given lytics for PE.  Don’t use it for patients who arrested from bleeding because hypothermia will result in coagulopathy.   In patients on Coumadin, you don’t need to reverse or correct their inr’s.  Plavix does not preclude therapeutic hypothermia.  Elise would also cool patients with hemophilia who were not actively bleeding.  Patients with risk for head bleed due to trauma need head ct prior to cooling.

PICIS has algorithm for hypothermia.  We will have to find out where algorithm will be stored in FIRST NET system.

There is some data to suggest that delay to cooling increases risk of death.  Minnesota study shows 20% increase in death with each hour of delay to starting cooling.

New study coming down the pike: Do we need to actually cool pt’s down to 33C or is 36C good enough? European study of 875 patients is looking at this question.

Post-arrest prognostication: You have to wait until 72 hours when using therapeutic hypothermia.  Cooling and associated meds can decrease brain function for 72 hours after ROSC.

Harwood question: what is the definition of coma in the post arrest patient?  Elise answer: If you give a verbal command with no response or GCS <8.

Can also use hypothermia in neonatal hypoxic ischemic encephalopathy. NNT=7 to reduce death or major neurodevelopmental disability. Can use in Pediatric Cardiac Arrest.

New research to use therapeutic hypothermia for traumatic brain injury.

Remember that there is usually a culprit coronary lesion with cardiac arrest. So patients should go to cath lab after resuscitated V-Fib arrest.

Use left femoral vein for cool guard catheter.

Sam Lam Question: What about patients that re-arrest? Elise answer: if patients re-arrest or require hi dose or multiple pressors then stop cooling.  Outcome is dismal.

SEE THE ACMC PROTOCOL ON THE NEXT PAGE

 

Levato   Febrile Neutropenia

Absolute neutrophil count less than 500 is neutropenia.  Temp>38 is a fever

ABx choices are Cefipime or Primaxin.   Vanco is limited to specific categories listed on form (shock, skin,foreign device infection, mucositis).  For beta lactam allergy: aztreonam/cipro/tobramycin. Pick 2 of these three.

Main concern in these patients is on gram neg infections.

 

Conference Notes 9-18-2012

Conference Notes 9-18-2012

Kutka/Urumov  Oral Boards

Case 1: Crytpococcal meningitis in a man with AIDs. Critical actions include ordering cryptococcal antigen and/or india ink test (usual csf studies won’t pick up crytpococcal infection), giving antifungal medication (amphotericin B), rewarming for hypothermia, get a head ct prior to LP.

Andrej’s comments: cryptococcal meningitis is rare if CD4 count is greater than 100.  Opening pressure can be high.  High opening pressure portends a worse prognosis.  Routine CSF studies can be completely normal so cryptococcal antigen and/or india ink studies are essential to making the diagnosis.  

Case2:  Intracranial hemorrhage.    Critical actions include airway management, treat BP, elevate head of bed, treat intracranial hypertension with mannitol.

Andrej’s comments: Treat BP if over 180 systolic with a iv drip antihypertensive medication.   Anti-seizure meds not routinely indicated.  Some weak data showing worsened outcome with seizure prophylaxis.  

Case 3:  7yo with BB gunshot to eye with a retro-orbital hematoma.  Critical actions include lateral canthotomy, consult optho, pain management and prophylactic antibiotics.

 Elise comment: Think about doing ct scan brain with iv contrast in patients with hx of cancer and immunocompromised patients.

Chastain    7-UP Scan for Hypotension

Non-invasive study to augment clinical evaluation in the hypotensive patient.

7 UP scan includes FAST plus lungs, aorta, parasternal echo views.

When in the sucostal window you should look at the heart for pericardial fluid, RV dilatation, and overall contractility.   Paradoxical movement of ventricular septum is also an indication of PE.  The septum should move from LV toward RV.  If it is moving from RV to LV think PE.  Also look at the cava to see if it collapses.  More than 50% collapse indicates a CVP less than 10.  You should take this measurement as close to the heart as possible.  A crude indicator of location is that you should be looking at the ivc right by the liver.

In PSL window if aortic root is more than 4cm think about dissection.    Be sure to image this window with enough depth to see the descending aorta.  Frequently pericardial fluid will collect in the pericardium anterior to the descending aorta.  

Lung windows can help with causes of dyspnea.   Use the linear probe in the 2nd and  3rd intercostal spaces bilat.  No sliding of pleura means pneumothorax.   Increased vertical comet tails (B lines) indicates chf.  Increase horizontal A lines indicates COPD or Asthma.

Apical view of heart can also show RV dilation and bowing of the septum to the the left.

LUQ view usually needs the probe to be “closer to the bed and closer to the head” than when viewing the RUQ.   The spleen/liver tends to be more superior/posterior than the liver/kidney. 

Wise   Deadly Triad in Trauma

Hypothermia/acidosis/coagulopathy comprise the deadly triad.

Damage control surgery is temporizing procedures to obtain hemostasis. 

Hypothermia: trauma causes loss of thermoregulation.  Hypothermia exacerbates coagulopathy by decreasing platelet activation and altering enzyme kinetics. It also alters fibrinolysis. 

Acidosis: Causes decreased contractility, vasodilation, an worsened coagulopathy.   Base deficit >6 and elevated lactate  correlates with increased mortality.

Fluids can cause dilutional coagulopathy and hypothermia.

2002 study with sick trauma patients:  coagulopthy required BOTH tissue injury and hypotension.  Protein C is over-activated in severe trauma that may be the mechanism of the coagulopathy.

Tranexamic Acid:  CRASH-2 trial showed all-cause mortality benefit.   Benefit in bleeding patients depended on time of administration.  First hour had most benefit.   After 3 hours may increase mortality.  

Elise comment:  ACMC does not have tranexemic acid currently.

Permissive hypotension: goal is to maintain BP only to the point of maintaining minimal adequate perfusion.   Resuscitation fluids/blood products are restricted.   Goal BP is 70-90 systolic.   Generally accepted that patients with penetrating trauma should not be resuscitated to normal BP prior to gaining hemostasis.

Damage control resuscitation: permissive hypotension  plus damage control surgery plus resuscitation volume is predominantly blood products rather than crystalloid.

Factor 7 was discredited as a resuscitation drug.   Erik and Elise pounded  Factor 7 into the ground during the discussion.     Erik said there is new data supporting further tranexemic acid. 

Recommendations: shoot for a lower BP goal, avoid large volumes of crystalloids, use the massive transfusion protocol, give TXA in the first hour, keep patients warm. 

TABLE 112-1   -- Categorization and Initial Treatment of Hemorrhagic Shock*

 

CLASS I

CLASS II

CLASS III

CLASS IV

Blood loss (mL)

≤750

750-1500

1500-2000

≥2000

Blood loss (% of blood volume)

≤15

15-30

30-40

≥40

Pulse rate

<100

>100

>120

≥140

Blood pressure

Normal

Normal

Decreased

Decreased

Capillary refill test

Normal

Positive

Positive

Positive

Respiratory rate

14-20

20-30

30-40

>35

Urine output (mL/hr)

≥30

20-30

5-15

Negligible

Mental status

Slightly anxious

Mildly anxious

Anxious and confused

Confused and lethargic

Fluid replacement (3:1 rule)

Crystalloid

Crystalloid

Crystalloid + blood

Crystalloid + blood

 

Chandra   Massive Transfusion Protocol (MTP)

Massive transfusion:  10 units of prbc’s in 24 hours,  or replacement of 50% of total blood volume in 4 hours.    Kids is >40ml/kg prbc’s  in 4 hours.

Who gets MTP?: ABC rule includes heart rate >120/bp<90/positive fast/penetrating mechanism.  More than 2 criteria activate MTP.    TASH score bp<100/hr>120/hgb<7/FAST/ Long bone fx/male gender.

1 unit of prbc’s increases hgb by 1.   Patients that receive FFP in a 1:1 ratio with prbc’s have a lower mortality.   Consensus for MTP is 1:1:1 ratio for prbc’s: platelets: ffp.   Battle field data from Iraq shows improved mortality with this ratio.

ACMC protocol: 10 units prbc’s, 6 units ffp, 1 unit aphoresis platelets, and 2 units of cryoprecipitate.  ER or  Trauma attending has to order this protocol.   The SYMS know how to order this. 

Goal of MTP: Map of 65 with adequate perfusion; basically bp of 80/60 with palpable pulses and warm extremities.    

Sam Lam comment: He questioned the component make up of the ACMC MTP because it is a little atypical based on trauma data in the literature.

OMI  Traumatic Brain Injury

Brain injury classification: Mild GCS=14-15    Mod GCS=9-13     severe GCS=3-8

Canadian head injury rule is a validated tool to identify patients at risk for positive ct or brain injury.

40% of moderate head injury patients(GCS9-13) have abnormal ct findings and 10% require surgery.

Severe head injury patients have the highest likelihood for brain injury and highest potential for benefit from surgery.   Get these patients to CT in 30minutes.   Tube all these patients.   Don’t routinely hyperventilate these patients.   PCO2 less than 25 increases mortality.

SAH from trauma are relatively benign.   Subdural hematomas tend to have significant underlying brain injury.   Epidurals often have little underlying brain injury.  Epidurals have great outcomes if manage properly.  Epidural hemorrhages may have a lucid period between initial loss of consciousness and later deterioration.    CT will be abnormal in these epidural cases from the time of initial injury.

Diffuse brain injury: concussion usually resolves in 6 hours.   Diffuse axonal injury due to shearing forces from high speed mvc’s.  Initial Ct will be normal despite severe coma.   Later CT’s or MRI will show punctuate hemorrhages.   Outcome for DAI is poor and any improvement takes months to years.

Monro-Kellie Doctrine: increasing volume inside fixed volume boney skull causes rapid increase in intracranial pressure.     

 

Cerebral perfusion pressure=MAP- ICP.   There is little data that medical care impacts outcome of brain injury.   Prevention of injury and prevention of secondary injury are keys to limiting morbidity/mortality.  Preventing hypoxia is probably the most important thing we can do to prevent secondary injury and  lower  patient mortality.   Hypotension is the second most powerful factor increasing mortality in brain injured patients.   After preventing hypoxia and hypotension, there is not much evidence that anything else helps outcome.

 ICP monitor:  keep ICP less than 20 and CPP>60. Improved outcome in patients who respond to hyperosmolar therapy.   This means mannitol at 1gm/kg.   Mechanism is osmotic mobilization of water across blood brain  barrier.  You can get hypotension from mannitol.   Hypertonic is an option for osmotic therapy that does not cause hypotension.

Some centers are studying hypothermia and suspended animation for severe brain injury.  The current thinking is that patients need low temp cooling for prolonged time (>48 hours).

Brain oxygenation are also being studied.

Hyperventillation:  works by dropping pco2 causing vasoconstriction.  This causes decreased brain blood flow and reduction in intracranial volume/pressure.  It can however cause brain ischemia.  So it has gone out of fmavor.

Kascia Nosek comment:  If patient is breathing over the vent settings, do you sedate them to avoid hyperventilation?   Dr. Omi,  yes.

Steroids: no benefit in brain injury.

Anticonvulsants:  Phenytoin reduces the incidence of seizures in the first week but not after.

Brett Negro comment: What are criteria for using steroids?   Dr. Omi,  patients with parenchymal brain injury including subdural hematomas should get phenytoin or phosphenytoin.  Keppra may be another option.   Anticonvulsants are all stopped at about a week.

Elise comment: Ketamine is probably a good choice for an induction agent in the hypotensive brain injured patient.   The risk of increasing intracranial pressure is low and it is less likely to cause more hypotension than etomidate.  

Conference Notes 9-11-2012

Conference Notes 9-11-2012

Kettaneh   5 Causes of ST Elevation

  1. STEMI
  2. Benign Early Repolarization: People under 50yo, J point notching, concave up ST elevation, Prominent T waves concordant with QRS
  3. Pericarditis: Diffuse STE and/or PR Depression,  Reciprocal changes only in AVR (ST depression and PR Elevation) 

For deciding between  Early repol and pericarditis: ST segment elevation  compared to T wave height ratio in V6 is greater in pericarditis  (STE height/T wave height).   The T wave in early repol is taller than in pericarditis and the ratio is lower in early repol.

  1. Bundle Branch Block.   Sgarbossa criteria: 1mm Concordant st elevation, 1mm Concordant st depression, discordant st elevation >/= to 5mm.  Cabrera’s sign: notching in S wave in V3-4.  Chapman’s sign notching in the R wave V6.
  2. LV Aneurysm: can lead to sudden cardiac death,  arrhythmia, thrombus.  Consider after MI, absence of hyperacute T waves.

 

Other causes: brugada, lvh, hyperkalemia, hypercalcemia, myocarditis.

 

Girzadas question: Is benign early repol actually benign?  Answer: there is controversy but most references feel it is benign .   Silverman comment: BER has an emerging literature that shows a possible risk of sudden cardiac death.   However, no one knows what to do with this EKG finding.  There is no treatment protocol currently for this.

Harwood comment: Pericarditis vs. Early repol use tp segment as your baseline for identifying PR depression.  For figuring out the ST to T wave ratio use the PR segment as your baseline.  PR depression boosts the  ST elevation part of the equation increasing the ratio in pericarditis.

 

 

Herrmann  5 Causes of Wide QRS

 

Harwood comment: The best lead to measure the width of the QRS is the lead with the widest QRS.

  1. Bundle Branch Blocks: QRS>120ms.  RBBB can be associated with heart disease and PE but can also be present in normal hearts.   RBBB in an acute MI confers increases mortality.   If wide and up in V1 it is RBBB.  If wide and down in V1 it is a LBBB.
  2. Ventricular Rhythms: PVC’s are common in nl hearts.   Rules of malignant pvc’s : frequent pvc’s, couplets/triplets, multiform, pvc on t wave.  Ventricular escape rhythms are another cause of wide qrs. Accelerated idioventricular rhythm is associated with reperfusion with TPA.
  3. TCA  Overdose:Look for wide QRS generally and tall/wide R wave in AVR
  4. Hyperkalemia: Always consider this if the QRS is wide.   The ekg may also show a slow rhythm with loss of p wave.
  5. WPW: Slurred upstroke of the QRS complex (delta wave) due to accessory pathway.  Delta wave widens the QRS and shortens the PR interval.

 

Barounis STEMI Conference

 

Case 1: LR’s for historical items indicative of AMI is  highest for radiation to both arms, radiation to right arm, diaphoresis, and radiation to left arm.   Pressure has a relatively low LR of 1.3.   chest pain that is reproducible has a LR of 0.4 which lowers the risk but does not make the risk 0.    When you don’t have an old ekg to compare with, make an old ekg by getting another ekg to look for evolution.

Comments: The ekg had subtle st depression in 1/AVL. Some subtle st segment straightening inferiorly.

2nd EKG was diagnostic for inferior STEMI (STE greatest in 3)  

Harwood/Silverman  Comment:  Gotta get a repeat EKGwithin 10 minutes.  MD may have to stay at bedside for 10 minutes to get another EKG in high risk patients.

Other guest comment: Women will present with symptoms that can be atypical.

 

Case 2:  Evolving Inferior MI.    Cardiology comments an evolving ekg with chest pain should go to cath lab.  Recent normal stress test does not preclude AMI.   PT should go to cath lab even if ekg improves with ntg if other ER ekg’s were concerning.   

Dr. Silverman comment: Don’t delay more than 3 minutes waiting for return call from patient’s primary cardiologist.  After a 3 minute delay gotta contact intervential cardiologist on call.  He felt safest option is to call STEMI first and after that attempt contact with the primary cardiologist.   That way you get both cardiologists as rapidly as possible.

 

Case3: EKG initially was non diagnostic in a young patient with chest pain.  Dr. Silverman  advised stat echo in this situation.   If echo is nl, ekg is likely not stemi but more likely BER.  If echo is abnormal, then pt should go to cath lab.

 

CT angio for CAD:  Cardiologists generally not for it due to radiation exposure and low sensitivity.  Dr. Trevedi did say it has a good negative predictive value.  Dr. Trevedi felt hypertensive patients with chest pain may be a good pt group to use this test.  It give info about aortic dissection in addition to showing the coronary arteries.

Mila Felder’s summary points:

  1. Repeat EKG in 10-15 min if questionable EKG and/ or persistant pain. EKG department and ED techs are accountable for giving it to physician to review. There is follow-up pending to making sure copies of EKGs are placed on the chart.
  2. In case of dynamic EKG and consistent story, activate code STEMI. During the day, the patient's cardiologist may be able to take them to lab if ready to go and able to get to the hospital immediately. Do not delay care/ cath for convenience, and ok to use interventionalist on call to avoid delay in door to cath.
  3. Pay attention to early repolarization (no longer considered benign). In case of consistent story, evaluating heart rate, other lead changes, potassium level, and other historical facts, be suspicious of early MI. Additionally, easy to miss the blocks in conduction when only looking for ST changes.

 

 

McKean  Syncope

 

Brugada:  Has been diagnosed in patients age 2-82. EKG findings can be transient. Pt’s have RBBB pattern with STE in septal leads.   Fever can bring out the findings.  Treatment is an AICD.

 

WPW: Treat with electricity for unstable patients and procainamide for stable patients.

Long QT syndrome: risk of polymorphic V-Tach.   Measure from start of Q to end of T. Quick and dirty is QT should be less than 0.5 the RR interval.   Treat with AICD.

 

HOCM: LVH without inciting stimulus.  Thickening of intraventricular septum.  You can get exertional syncope due to dynamic  LV outflow changes.   On EKG pt’s will have LVH and deep narrow q waves V4-V6.  Treatment is myomectomy and pacer/aicd

 

PE: Sinus tach is most common EKG abnormality.  Also look for RV strain pattern (t wave inversions inferior and anterior-most specific finding).   Pt’s may have RBBB.   S1Q3T3 is nonspecific.

 

A number of ekg examples were discussed along with some embarrassing old pictures of residents especially Barounis.

 

Harwood pimp question: What is LGL?  Brian Febbo knew it is WPW with no delta wave.  Lown-Ganong-Levine Syndrome is diagnosed by the presence of a short PR interval and normal QRS complex on the surface electrocardiogram (ECG).

 

Kessen  Heart Blocks

 

Sorry I missed part of this lecture.

Lenegre’s DZ:  fibrotic sclerodegenerative change of conduction system progresses to complete heart block

Lev’s DZ: sclerosis of left side of heart in older patients causing heart block

 

1st degree av block: PR interval >0.2msec=5 small boxes.

 

2nd degree AV block Type 1=Wenckebach.  Progressively longer pr intervals.  RR interval shortens until the qrs gets dropped.   Not treatment indicated.

 

2nd degree AV block Type 2: PR interval remains constant before and after non-conducted atrial beat.   Atrial rhythm is regular and ventricular rhythm is irregular.

 

Look for AV block with inferior MI’s.

 

Harwood comment:  SA block is uncommon but it happens.  You can only see type 2 Sinus block on an ekg.  You can’t see Type 1 or 3 SA block.   SA block is different than AV block. 

Second degree SA nodal exit block has two types.

  • § 
  • §  In type II exit block, the P-P output is an integer multiple of the presumed sinus pacemaker input

Sayger/Felder/Katiyar /McGurk   Billing and Coding

 

All pneumonia patients going to ICU require blood cultures before antibiotics.

You need 10 ROS systems for level 5.

Document that you visualized and  interpreted the xray and give your interpretation.

Document the number of and type and drug that you used for nebulizer treatment.

You need either a social or family hx to get a level 5.

Keep track of your time you spend with critically ill patients.   Any time the attending spends on the care/ordering/discussion/documentation/decision making with the critically ill patient should be counted toward critical care time.

You need 8 organ systems on physical exam to bill a level 5.

Mnemonic: FORTUNATE    4-2-10-8.  4 HPI items, 2 history items, 10 ROS items and 8 physical exam items to bill level 5.

 

Ryan  Medical Student Review     Confidential Meeting

 

 

Conference Notes 8-28-2012

Conference Notes 8-28-2012

Grippo  Ortho Jeopardy

Perilunate Dilocation: Look at the lateral view of the wrist.   The capitate/lunate/radius need to line up.  If the capitates is dorsal to the lunate the dlx is perilunate.   If the lunate is dislocated volarly, it is a lunate dlx.

Supracondylar FX: 60% of peds elbow fxs.   Severe fractures that are not treated properly can develop Volkman’s ischemic contracture.

Femoral neck fractures: Have risk of avascular necrosis.   Older patients will get a arthroplasty.

Scapholunate DLX:  widening of space between scaphoid and lunate.  Terry Thomas sign.  Treat with radial splint.

Montaggia Fx:  Proximal ulnar fx with dislocation of radial head.

Galeazzi Fx:  Distal radius fx with dislocation of radio-ulnar joint.    Mnemonic is MUGR=montaggia/ulnar fx galeazzi/radial fx

Bennet’s Fx: Intra-articular fx at base of thumb. 

Barton’s Fx: Distal radius fx with intra-articular involvement.  Usually fx goes thru volar aspect of radius.  Can have either dorsal or volar angulation.

Bohler’s Angle: Normal is greater than 20 degrees.   If less than 20 degrees that is indicative of a calcaneous fracture.    With calcaneal fractures check for compartment syndrome in foot.  Also look for other joint and spinal injuries in patients who fell from height.

Lis Franc Fx:  tarso-metatarsal FX/DLX.  Look for fx at base of 2nd mt and/or non-allignment of based of second MT and middle cuneiform.

Boxer’s fx: Needs reduction if angulation >40 degrees.  If pt has associated fight bite give antibiotic prophylaxis.    After reduction place in ulnar gutter splint with finger in flexion.

Lovell comment: Frequently fight bite injuries require OR irrigation and debridement.   Harwood added that in the OR it can be determined whether the bite went into the joint space.    Both felt IV antibiotics were indicated and hand consult for either OR or Obs admit or Very close follow up.   This is high risk medico-legal situation.

Jone’s Fx:  Fx of metaphysis/diaphysis junction of 5th MT.  Risk of nonunion.  Non-weight bearing for 6 weeks.   Needs Ortho follow-up.  Psuedo-Jones Fx is basically an avulsion fx of tuberosity of 5th MT.   These heal well and only require cast shoe.

Salter Harris FX:  Type 2 is most common.    Mnemonic is ME: metaphysis involvement is a  2, epiphysis involvement is a 3.  1 is easy to remember because it is just thru the physis and 4 is also easy because it goes thru both the metaphysic and epiphysis.   5 is a compression injury to the physis.  

Pilon Fx: Bad comminuted distal tibial fx due to talus ramming into tibial plafond due to a fall from height. 

Chauffeur’s(Hutchinson’s)  Fx:  Fx of the radial styloid.   Used to occur when turning the crank of early model  cars.  

Barounis     Undifferentiated Shock

Shock: Inadequate O2 delivery to meet tissue demand.

Oxygen delivery=(HR x SV) x 1.34 X HGB X SAO2 X10.    HGB and O2 Sat are the most important factors for O2 delivery.

Shock is bad because it results in anaerobic metabolism and lactate production.   The sodium potassium pump malfunctions.  Lactate is the cry of poorly perfused mitochondria.

Types of shock: 

Obstructive (tamponade/tension pneumo/pe/auto peep/rv infarct) Eval for this is to listen to breath sounds, use ultrasound.  Check EKG for signs of RV infarct; lead III will have more st elevation than lead II. Check for auto-peep on vent.

Distributive shock: (sepsis/cyanide/anaphylaxis)  Bounding pulse with hypotension.

Cardiogenic Shock: cool clammy, altered mentation

Hemorrhagic shock:  the patient is bleeding.

Approach to shock: Assess heart rate (pulse is not the main issue between 60 and 180),  make a volume assessment/obstructive assessment (cvp/U/S of VC /urine output/gingival mucosa),  assess contractility with U/S,  figure out the SVR (check extremities for warmth/bounding pulse/coolness/decrease pulse)

On ultrasound if IVC collapses more than 50% with inspiration the patient is volume responsive.  This assessment is obtained with the subcostal long view of aorta.   You also want to check the abdominal aorta/pericardium/rv /morrison’s pouch.  

History is unreliable in the assessment of shock.   Physical findings are more reliable than history.

PEEP helps push fluid out of the lungs into the right heart.  Also the increased thoracic pressure from PEEP helps move blood to abdominal organs/brain/extremities by pressure gradient.

Jim Jensen  PharmD   Vasopressor Review

Dopamine:  Indications septic shock, hypotension without hypovolemia, symptomatic bradycardia.  Can cause arrhythmias.

Levophed: More potent alpha agonist.   Indicated for septic shock or hypotension due to low svr.  Increases myocardial oxygen demand, may cause arrhythmias. 

Phenylepherine: Soley an alpha agonist with no beta effects.   Last line pressor .  Start high dose and titrate down because it is a relatively weak vasopressor.   Harwood comment: Only use for this agent is  neurogenic shock.     

Epinepherine: Mixed alpha and beta agonist.   Indicated in ACLS, septic shock after dopamine or norepi, anaphylactic shock.

Vasopressin: Smooth muscle vasoconstriction.  Inidcated in ACLS and is an option in septic shock with catecholamine resistance.   

If a vasopressor extravasates out of the vessel, you can use phentolamine locally to counteract the effects of the vasopressor.   Harwood comment: Give the phentolamine through the IV that extravasated so that the antidote goes right to where the  tissue injury has occurred.

Central line is required for Epinepherine drip and norepi drip.   Central line not required for dopamine, phenylephrine, vasopressin.

Dobutamine:Beta agonist that  increases cardiac contractility/cardiac output and vasodiates.   Can cause arrhythmias and hypotension.

Milrinone: Phosphodiesterase inhibitor increasing CAMP.  Increases cardiac output but does cause vasodilation. 

Plavix vs. Ticagrelor (Effient):  Ticagrelor has a stronger antiplatelet effect and has been shown to reduce thrombotic events compared to Plavix.  This comes with the cost of higher rate of  bleeding.  ASA dosing over 81mg decreases the effectiveness of Ticagrelor.

Carlson  Toxicology Antidotes

Antidote: Any treatment that lowers the LD50 of a toxin.  Direct antidotes act right at the site of the toxins action.    Indirect antidotes are supportive such as cooling, oxygenation, folate co-factor replacement etc.

Fomepizole is antidote for toxic alcohols.  Blocks alcohol dehydrogenase.   Pyridoxine is co-factor antidote for ethylene glycol.   Folate is a co-factor indirect antidote for methanol.

Lead poisioning:  Antidotes are succimer, BAL, EDTA.  In severe cases use BAL and EDTA both.   Can’t give BAL to patients with peanut allergy.

Mushroom poisoning with seizure:  Antidote is pyridoxine for gyromitra poisioning.    Gyromitra acts similar to INH and blocks GABA production.  

Clonidine poisoning: Antidote is narcan.  May need higher dose.  Repeated 2mg doses up to 10 mg.  There is controversy about the effectiveness of this antidote.

Hydrogen sulfide poisoning: Antidote is sodium nitrite for the sulfhemoglobinemia.   HBO is a second line direct antidote for this as well.

Calcium channel blocker OD: First line tx for severe OD’s is Insulin 0.5U/kg bolus followed by 0.5U/kg/hr drip and supplemental glucose therapy.

Anticholinergic toxidrome: Antidote is benzodiazepines first line.   Physostigmine is a direct antidote that should only be used with caution.  There is EM literature that shows physostigmine is actually relatively safe in patients with clear cut anticholinergic symptoms without other coingested substances.  The problem is that clear cut isolated anticholinergic OD’s are not very common.   

 Paraquat: Antidotes are Fuller’s earth, bentonite.  Don’t give O2 because it will cause pulmonary fibrosis.  

Coral snake: Red on yellow, Kill a fellow.   Coral snake (elapid) antivenin.    Red on black, venom lack refers to a non-venomous milk snake.   

Sulfonylurea overdose: Antidote is glucose and  octreotide.

Methylene Chloride:  Methylene chloride is broken down to CO in the liver.   Treat with HBO.  Methylene choloride has a long duration of action so patients may need multiple dives.

 Lily of the valley, fox glove,and oleander are plant sources of cardiac glycosides (digoxin): Antidote is digibind.   Atropine can also work by reducing vagal tone.

Rattle snake bite: Antidote for crotalid bites is crotalid antivenin.  Indications for antivenin are local spread,  coagulopathy , abnormal vitals.   Mnemonic: Spread, bled, almost dead.   Give 5 vials minimum.  Be prepared to manage anaphylaxis.

Hydroflouric acid:  Treat with calcium gluconate.   Don’t use calcium chloride because it can cause tissue damage.  Pt will have a lot of pain.  Can give calcium gluconate via topical gel, local injection, and intra-arterial infusion.   Needs hand consult or transfer to burn center.

Methemoglobinemia:  Treat with methylene blue.   HIV patients with G6PD deficiency  on dapsone for PCP can develop methemoglobinemia.

Amanita Phylloides muchroom.  Will cause vomiting more than 6 hours after ingestion.  Amanita acts like amped up apap resulting in centrilobular necrosis of liver.   Antidote is nac.

Organophophates: Treat with atropine and 2-PAM.   If you have tachycardia with cholinergic OD think hypoxia as secondary to pulmonary/airway secretions.   Still need to give atropine. 2-PAM regenerates acetylcholinesterace.

Willison/Carlson   Oral Boards

Case 1. Transverse myelitis.  Critical actions were perform detailed neuro exam, rule out cord impingement with mri, foley decompression of bladder.  Triad of sudden onset back pain, sensory changes (including allodynia) and weakness/sphincter dysfunction.   #1 thing for emergency physician to do is rule out cord compression.   Most references advise steroid treatment.   Can be a harbinger of MS or sarcoid.  

Case2. Depakote(valproic acid)  overdose with severely high ammonia level.  Critical actions were  intubation, check valproic acid and ammonia levels, treat with L-carnitine.  Can dialyze for severe cases.   Metabolism of depakote requires carnitine.  When you use up your carnitine you produce the toxic metabolite ammonia.  Giving l-carnitine allows normal metabolism of depakote.

Case3. Molar Pregnancy.  Critical actions were give iv fluids, get beta hcg, get u/s and identify molar pregnancy, consult ob-gyne.  Molar pregnancies occur 1 in 1200 pregnancies. Increased risk at extremes of age.  Two types genetically 69xxx or 69xxy or 46xx or 46 xy.  There is a chance of malignancy in both types.  Worst outcome is with patients that present with lung mets.  BHCG is usually great than 100,000. Uterus is larger than expected for age.  

Conference Notes 8-21-2012

Conference Notes  8-21-2012

Schwab/Barounis    Oral Boards

Case 1.  Toxic Alcohol-Methanol ingestion.  Recognize anion gap acidosis.  Calculate osmolal gap. Give Fomepizole.   Arrange for hemodialysis.    Pt required intubation to protect airway.   Bicarb drip may be used for acidosis but it is not a critical action.

Case 2.  Anterior Shoulder Dislocation.   Perform neuro-vascualar exam of injured extremity.  Give procedural sedation or intra-articular anesthetic.  Use any described reduction technique.

Case 3.   Retropharyngeal Abscess.  Identify pre-spinal soft tissue swelling. >7mm at C2 or >14 at C6 is abnormal.  The pre-vertebral soft tissue width should not exceed the width of the vertebral bodies.  CT of neck will give more detail of soft tissues than plain radiograph.    Give appropriate IV antibiotics.  Intubation is rarely required unless patient  looks very sick and is planned to be transferred.  Surgical airway may  rarely be required.

E Kulstad   Work up for PE

The prevailing thought is that we try to identify PE’s to save someone’s life.   This idea is based on older data that found PE’s to have a reasonably high mortality.  Current data from Jeff Kline 2008 shows that in 13 ED’s in the  US and NZ the overall PE mortality is 0.2% (13/8138).

Is PE mortality lower today because of better treatment?  There is only 1 controlled trial of anticoagulation for venous thromboembolism.   This one study showed no treatment difference between heparin and ibuprofen.   The thought is that mortality is better today due to emergency physicians casting such a wide net that we are identifying small clinically insignificant PE’s.  That broader group has a much lower overall mortality.

Small peripheral PE’s pose an unknown threat.   Small clots may be transient and normal.   If we scanned everyone in the audience, we would find a few small PE’s.  One study showed a 20% rate of PE in autopsies for persons killed instantly by a traumatic accident.

When using a low specificity test in a population with a low prevalence of disease (ie. CT for PE in low risk patients) false positives exceed true positives.   The PERC study showed a 7% prevalence of PE based on imaging.   Probably many (most?) of these positive scans were false positives.   To make matters worse inter-rater reliability between radiologists reviewing scans to identify PE is not very good.  The more likely prevalence based on calculations Erik walked us through is 2.3%.

Assume there is an 80% reduction in mortality of PE due to heparinization.  This is likely a gross overestimation of treatment effect.  Erik then walked us through calculations of harm and benefit of identifying and treating PE.  Risks of harm include renal injury, cancer risks, risks of hemorrhage. The final calculations show that work up and treatment for PE causes more harm than benefit for patients.   These calculatons  use conservative estimates of harm and generous estimates of benefit.   The conclusion  is that current practice of working up PE’s  has 6X greater chance of harm than benefit.

In the US standard of care probably forces us to persist in working up patients for PE.

Lovell comment:  Can we use normal vitals to not pursue a work up?    Can we use a higher d-dimer cut off for low risk patients?    Erik responded yes to both.    You can use a double of the standard d-dimer cut off for low risk patients.

Barounis comment:   He got a response from the author Dave Neuman that pt’s with a Well’s score less than 2 need no further work up .

Gourineni    Peds Ortho

If pt has limb ischemia due to a fx or dlx you should immediately attempt reduction.   Then consult both Ortho and Vascular Surgery.   Don’t allow the child to eat or drink  in the ER if there is any chance of patient going to OR.

Compartment  syndrome:  Gourineni  feels compartment pressure measurements are not accurate.  He prefers the symptom of muscle pain and sign of tense compartment.    He also likes the delta pressure which is the difference between diastolic blood pressure and compartment pressure.  Pain with passive movement is also a sign he favors.   If you suspect compartment syndrome call both the Ortho resident and Ortho Attending.    Keep limb at heart level, remove any bandages, reduce any deformity.   These patients require surgery in 3-4 hours.

Open fractures: Early antibiotics with ancef is more important than timing of surgical debridement.  Open fractures of hand do not require surgery.  Irrigation and antibiotics in the ER is adequate for hand or  distal extremity open fracutes.

Dislocations:  All dislocations need to be reduced in ER.   Delay in reduction in elbow/knee/ankle/foot will result in ischemic injury.   40% risk of posterior tibial artery injury in knee dislocations.   Make sure joint has good range of motion after reduction.  If it doesn’t, Dr. Gourineni wants to know about it.

Fractures: Boney deformity tends to straighten out.   Deformity does not improve around elbow.  So, all displaced elbow fractures require ORIF.    Splints should be long for supracondylar fx’s to proximal humerus.    Femur fractures need a splint extending up to chest wall.

Clavicle: most clavicle fx’s are treated non operatively.   Surgery is required for skin tenting or posterior sterno-clavicular dislocation.

Proximal humerus fx: 100% displacement and 1cm of shortening will spontaneously remodal.  This is due to majority of bone growth at proximal humerus.

Supracondylar Fx:  Look at anterior humeral line.  If it bisects the condyle there is minimal displacement.  These patients can be splinted and discharged with close ortho follow up. If the condyle is posterior to anterior humeral line there is significant displacement and pt should be admitted for surgery.   If there is vascular compromise, pt will go to OR in a few hours.  Splints should not be at 90 degree flexion.  30-45 degrees  is better.   Check interosseus nerve and radial nerve function with thumb IP flex/extension or OK sign.

  Monteggia Fx-Dlx:  Think of this any time you see a proximal ulnar fx.   It is the combination of proximal unlar fx and radial head dislocation.   If the radial head doesn’t line up with the capitellum it is dislocation.

 Elbow dislocation:  For all elbow dislocations do the Roberts maneuver.  Extend and supinate wrist to remove any boney particles in elbow joint.

Displaced distal forearm fractures will frequently heal and remodel in 2-3 months.  You don’t need to reduce most of these.   If parents want it reduced and you feel you can reduce it, it is ok to attempt reduction.

Any hip pain should initiate an Ortho consult.

MCP dislocations that are angulated not parallel to bone should be reduced by not pulling the digit but  rather pushing the digit closer to the metacarpal bone and sliding the digit back into place.

Lis Franc: If patient has tenderness with torsion of forefoot. Get an xray looking for fx of prox 2nd metatarsal or non-allignment of middle cuneiform and 2nd metatarsal.  Either way if xray is nl or abnormal splint patient and keep them non-weight bearing with follow up in Ortho clinic.

Joint Aspiration and Reduction Clinic

 

 

Conference Notes 8-7-2012

Conference Notes  8-7-2012

Grippo/Lovell  Oral Boards

Case 1:  Central Cord Syndrome:  Treat with application of cervical collar,  ct the cspine for evaluation for fracture.  Recognize weakness in upper extremities.  MRI to evaluate the spinal cord.  Steroids for this injury is controversial.   Consult neurosurgery.  ICU admit.   Identify urinary retention.   Central cord is the most common incomplete spinal cord injury.   Classic case is old person/hyperextension injury/arms weaker than legs.   Check for pain and temp perception in suspected cord injuries.

Case 2: Cardiogenic Shock:  Patient with Hypotension and hypoperfusion with AMI. Treat with BIPAP or intubation.  Diagnose STEMI.  Support hemodynamics with inotrope and pressor (dobutamine/dopamine).  Cardiovert unstable VT.  Get to the cath lab.   These patients look very sick and may have altered mental status.

Case 3: Nasal Foreign Body: Treat with parent giving forceful breath into patient’s mouth.   Works 50% of the time with non-sharp edged objects.   Multiple other techniques are available to remove the FB (suction, fogarty catheter).  Button batteries in nose can cause necrosis.

Harwood comment:  Best approach to the steroid issue is discuss that it is controversial and then either give or not give.   Probably better to intubate the patient so they can be more safely managed in the cath lab.  

BINGISSER   Geriatric EM

Dr. Bingisser is a practicing EP in Switzerland. ED’s in Switzerland also have crowding issues.

Seniors take taxis to the ER because it is cheaper than an ambulance.   The Rolling Stones took along a geriatrician on their last European tour.

Problems managing elderly patients: poor communication/atypical presentations/broad spectrum of illness/complex interaction of social/medical problems/non-specific complaints/subtle vital sign changes to serious illness.

Triage in the elderly is difficult for the above reasons.  Elderly patients are commonly undertriaged.  Vital sign abnormalities were commonly unrecognized. Also high risk situations are frequently unrecognized.

Localized weakness: 75% were strokes 25% were stroke mimics.    Genralized weakness complaints turned out to include diagnoses from all ICD9 code  chapters.

Non-specific complaints:  1 year mortality for elderly patients with nonspecific complaints in Dr. Bingisser’s study was 13.5%.  30 day mortality was 6.4%.   In 1210 patients, they made over 300 different diagnoses!  Uti was most common cause and over 50% of those also had sepsis.    50% of ED diagnoses for non-specific complaints were incorrect.   6 predictors for serious outcomes of elderly patients with nonspecific symptoms  are elevated BUN,  low sodium,  elevated CRP, history of exhaustion, clinician gestault, chf.

Viswanathan/DKA

I missed this lecture giving Dr. Bingisser a tour of our ER,  sorry.  But, I did hear,” don’t bolus insulin or Dr. V. will hurt you”.

Roy  Peds Vignettes

Case 1: Lethargic 6 month Infant, ddx includes CNS/tox/sepsis/metabolic/trauma/hypoglycemia/inborn errors/intussusception.    Toxic encephalopathy can include hypertensive encephalopathy in kids due to post-strep glomerulonphritis.   MCAD is a substrate dependent inborn error of metabolism that presents as hypoglycemia when a child doesn’t eat as regularly as normal due to an illness or sleeping longer. Unexplained neuro symptoms in an infant, you should think GI process.   Think shigella in a febrile infant with diarrhea and seizure.   Classic case of intussusceptions is lethargic kid in second half of first year of life.  KUB in intussusceptions may show paucity of gas on right side.    Intussusception used to be uncommon in kids under 4 months.  However, now with rota virus vaccine it is possible under 4 months.  Dr. Roy has seen 5 cases in the last two years in kids under 4 months.  If the child has not had a rota  virus vaccine, it is unlikely to get intussusceptions under 4 months of age.    HUS is another cause of lethargy and seizures in an infant.   Think HUS in a kid with gastro that got better then gets sick again 1-2 days later.   Check a CBC in a gastro kid who has had a course of illness of 4-5 days to look for low platelets or hemolysis/anemia.  CBC findings will precede bun/cr changes.  Dr. Roy makes a point about  the change in color in kids with intussusceptions or HUS.  Kids can also get HUS from pneumococcus.

Unexplained respiratory symptoms in an infant think: Heart-CHF (check the liver for swelling).  Myocarditis clues are marked tachycardia, tachypnea, murmur.   GERD.   Upper airway obstruction such as laryngo-tracheomalacia/sub-glottic stenosis/croup is unlikely in a young infant

Xrays are not necessary in most  asthmatics or most simple croupers.   Epiglotitis doesn’t bark like a seal.  They usually have muffled voice and are drooling because swallowing is painful.

Nausea/vomit/abdominal pain without fever or diarrhea is DKA until proven otherwise.

Fever for 5 days is Kawasaki’s until proven otherwise.

Puffy eyes and puffy hands in kids is a renal problem until proven otherwise.

Do a CBC in a limping kid to eval  for leukemia.   1 out of 7 kids with new onset leukemia presents with musculoskeletal pain.  Don’t believe the parents’ story of trauma.

Case 2/3: 3 week old infants with vomiting.  Think pyloric stenosis.  Olive mass in ruq is uncommon.  If child has low sodium and high potassium think congenital adrenal hyperplasia.  Look for hyperpigmented scrotal skin in kids suspected of congenital adrenal hyperplasia.

Sickle rules: fever warrants admission.   Get a retic count to r/o aplastic crisis.   Respiratory or chest complaints require a CXR for acute chest syndrome.  Check spleen for sequestration crisis.  Most of the sequestration crises at ACMC over the last decade have been kids over the age of 10.

Grippo   ACLS Update

Switch out your persons doing compressions every 2 minutes.    Avoid over-bagging.

Defib with 200J biphasic.

1mg epi or 40u of vasopressin

Amiodarone 300mg IV

PEA: consider causes like pneumothorax, hyperkalemia

Bradycardia: Atropine 0.5mg to 3mg max.   Transcutaneous pacing.  If you can’t capture with TCP,try epi drip or dopamine drip.  Last line is transvenous pacing.

Wide Complex Tachycardia: If unstable with pulse cardiovert.  If no pulse defibrillate.

Narrow complex tachycardia: SVT/AFIB with RVR/AFutter/MAT

Wide and Irregular:  WPW with AFIB,   AFIB with BBB

Harwood comment: Use your right hand to help differentiate RBBB  and VT.  In RBBB second rabbit ear should be taller like your right hand with the 3rd finger taller than the index finger.  If the first rabbit ear is taller, it is more likely to be V-tach.

 

Conference Notes 7-31-2012

CONFERENCE NOTES 7-31-2012

CHASTAIN/GIRZADAS  ORAL BOARDS

n  CASE 1: Ethylene glycol poisoning. 

n  Intubate (PC)

n  Treat with 4-Methyl Pyrazole ( fomepizole 15 mg/kg) or ETOH (PC,MK)

n  Arrange Hemodialysis (MK,SBP)

n  Give Antibiotics for aspiration pneumonia (MK, PC)

n  Anion Gap= Na-(CL + HCO3)<15

n  Osmolal Gap= 2X Na + Glucose/18(20) + BUN/2.8(3) + ETOH/4.6(5)

n  Measured Value - Calculated Value <20

CASE 2: Femoral artery injury due to GSW

n  IV fluid bolus 20ml/kg (PC)

n  PRBC transfusion 15ml/kg (MK, SBP)

n  Identify Hard Signs of vascular injury and perform ankle/brachial index (MK)

n  Trauma/Vascular surgery consultation (SBP, PC)

n  Hard signs of vascular injury mandate angiography or surgery.

n  Pulsatile bleeding

n  Pulsatile hematoma

n  Bruit/thrill

n  Absent/diminished pulse distal to injury=ABI<0.9

n  Ischemic signs (pain, pallor, coolness, paralysis)

 

CASE 3: Pyloric Stenosis

n  Consider Pyloric Stenosis (pmh, olive, BMP, U/S, Upper GI) (PC,MK)

n  IV hydration (20ml/kg)

n  Admission for planned surgery (SBP)

n  Hypo-chloremic/kalemic/natremic metabolic alkalosis

n  Today most are diagnosed prior to electrolyte abnormalities

n  Males more common 5:1

n  Associated with macrolide antibiotics

n  Laproscopic Pyloromyotomy

 

WOOD   MEDICAL-LEGAL ASPECTS OF EM

Philosophy: the study of questions that can’t be answered.  Religion:the study of answers that can’t be questioned.

Case 1 Chronically ill elderly patient with hypoxia.  Husband wants everything done.   Autonomy is the primary ethical rule but pt’s have to have understanding.  If pt can’t make decision.  There is a principal of implied consent in situations of emergency.  Husband has the power to decide for wife.  But what if husband is demanding futile care?  Decision making can only be taken away from husband if he lacks decisional capacity.  Answer is to inform husand that further work up/treatment may worsen suffering and palliative care may lessen suffering.    It is ok to discontinue ventilator.  There is no distinction between action and inaction.  Taking patient off ventilator is acceptable.  It is ok to give small doses of opioids/benzodiazepines to relieve suffering.  Don’t give a large dose that could kill a terminal patient.  Document how the patient looks and document your intent to relieve suffering.

Case 2  Man found unresponsive in his car. He is a nurse who is abusing oxycontin.  It is ok for doctors to speak with other MD’s if it is in the context of treating the patient.  Confidentiality is well supported by law and custom but it is not absolute.  Exceptions exist due to societal interest.  Doctors are required to report child abuse or turn in weapons.    Doctor-patient relationship is much less protected legally than attorney-client relationship.  The doc has a duty to turn drugs (evidence) over to the police or security. You are more likely to get sued for not reporting something than for reporting something.

How do you determine decisional capacity.  If you have a 0.8 etoh level, there is a legal presumption that you have lost the psycho-motor skill to operate a car. There is NOT the legal presumption that a patient gives up all their rights to leave the ED as long as they are not driving.   An etoh level above 0.8 is not grounds alone to forcibly restrain a patient from leaving the ED.  

Case 3   16yo female who had sexual encounter.  Doctor refused to give post-coital contraception based on his own moral code.  There is no statute in any state that says you must be 18yo to consent for medical care.  No doctor has ever been successfully sued for non-negligent treatment of kids over 14 without consent from parents.   32 states have a statute saying it is ok for docs or pharmacists to not tell a patient about contraceptive/abortion options if it is against their conscience.   If a patient asks the doc if there is a morning after pill option for her, the doc can’t lie and say there isn’t.  

Dr. Woods Notes: Notes and Bibliography

 

Legal and Ethical Issues in Clinical practice.

 

Joseph P. Wood, M.D.,J.D

Principles of medical ethics

A physician shall be dedicated to providing competent medical care, with compassion and respect for human dignity and rights.

A physician shall uphold the standards of professionalism, be honest in all professional interactions, and strive to report physicians deficient in character or competence, or engaging in fraud or deception, to appropriate entities.

A physician shall respect the law and also recognize a responsibility to seek changes in those requirements which are contrary to the best interests of the patient.

A physician shall respect the rights of patients, colleagues, and other health professionals, and shall safeguard patient confidences and privacy within the constraints of the law.

A physician shall continue to study, apply, and advance scientific knowledge, maintain a commitment to medical education, make relevant information available to patients, colleagues, and the public, obtain consultation, and use the talents of other health professionals when indicated.

A physician shall, in the provision of appropriate patient care, except in emergencies, be free to choose whom to serve, with whom to associate, and the environment in which to provide medical care.

A physician shall recognize a responsibility to participate in activities contributing to the improvement of the community and the betterment of public health.

A physician shall, while caring for a patient, regard responsibility to the patient as paramount.

A physician shall support access to medical care for all people.

 

Adopted by the AMA's House of Delegates June 17, 2001.

Case of Patient in a Vegetative State

Medical definition

Any person with an illness that is not able to function properly without artificial help.

Legal/ethical definition

As opposed to brain death and comatose, PVS is not recognized as death in any legal system. This ethical grey area has led to several court cases involving people in a PVS, those who believe that they should be allowed to die, and those who are equally determined that, if recovery is possible, care should continue. This ethical issue raises questions about autonomy, quality of life, appropriate use of resources, the wishes of family members, professional responsibilities, and many more.

History

The syndrome was first described in 1940 by Ernst Kretschmer who called it apallic Syndrome.[1] The term persistent vegetative state was coined in 1972 by Scottish spinal surgeon Bryan Jennett and American neurologist Fred Plum to describe a syndrome that seemed to have been made possible by medicine's increased capacities to keep patients' bodies alive.[2][3]

 

[edit]

Classification

Terminology in this area is somewhat confused. While the term persistent vegetative state is the most frequent in media usage and legal provisions, it is discouraged by neurologists, who favour the terminology of the Royal College of Physicians (RCP) which refers only to the vegetative state, the continuing vegetative state, and the permanent vegetative state.[4]

The vegetative state is a chronic or long-term condition. This condition differs from a persistent vegetative state (PVS, a state of coma that lacks both awareness and wakefulness) since patients have awakened from coma, but still have not regained awareness. In the vegetative state patients can open their eyelids occasionally and demonstrate sleep-wake cycles. They also completely lack cognitive function. The vegetative state is also called coma vigil. The continuing vegetative state describes a patient's diagnosis prior to confirmation of the permanence of the condition. The permanent vegetative state occurs when the vegetative state is deemed permanent; a prediction is being made that the patient will never recover awareness. This prediction cannot be made with absolute certainty. However, the chances of regaining awareness diminish considerably as the time spent in the vegetative state increases (Royal College of Physicians, 1996).

This typology distinguishes various stages of the condition rather than using one term for them all. In his most recent book The Vegetative State, Jennett himself adopts this usage, on the grounds that "the 'persistent' component of this term ... may seem to suggest irreversibility".[2] The Australian National Health and Medical Research Council has suggested "post coma unresponsiveness" as an alternative term.[5]

 

[edit]

Signs and symptoms

Most PVS patients are unresponsive to external stimuli and their conditions are associated with different levels of consciousness. Some level of consciousness means a person can still respond, in varying degrees, to stimulation. A person in a coma, however, cannot. In addition, PVS patients often open their eyes, whereas patients in a coma subsist with their eyes closed (Emmett, 1989).

PVS patients' eyes might be in a relatively fixed position, or track moving objects, or move in a disconjugate (i.e. completely unsynchronized) manner. They may experience sleep-wake cycles, or be in a state of chronic wakefulness. They may exhibit some behaviors that can be construed as arising from partial consciousness, such as grinding their teeth, swallowing, smiling, shedding tears, grunting, moaning, or screaming without any apparent external stimulus.

Individuals in PVS are seldom on any life-sustaining equipment other than a feeding tube because the brainstem, the center of vegetative functions (such as heart rate and rhythm, respiration, gastrointestinal activity), is relatively intact (Emmett, 1989).

 (Available on Wikipedia.org with essentially no copyright restrictions).

 

Conflict with Surrogate Decision-Maker

 

1)       Baruch Brody, Special Ethical Issues in the Management of PVS Patients, 20 L., Med. And healthcare 104 (1992)

2)       In Re Wanglie, No. PX-91-283 (Minn.D.Ct. June 28, 1991)

(Hospital sought order to replace Husband as the surrogate decision-maker. Court did not address the substance of the decisions made by the Husband finding that the only materially relevant question was whether the Husband was in the best position to know what the patient would want done if she was able to speak for herself).

3)       “The Physician-Surrogate Relationship” Archives of Internal medicine, June 11, 2007

4)       “Time to Move Advance Care Planning Beyond Advance Directives” Chest 2000

 

May a Physician Sedate a Terminally ill Patient to the Point of Unconsciousness?

 

5)       Vacco v. Quill, 117 S. CT. 22293 (US 1997)

 

6)     Quill, T. E., Byock, I. R., for the ACP-ASIM End-of-Life Care Consensus Panel. Responding to

intractable terminal suffering: the role of terminal sedation and voluntary refusal of food and fluids.

Ann Intern Med. 2000;132:408-414.

 

7)       See CEJA Report 5-A-08 Referred to Reference Committee on Amendments to Constitution and Bylaws (Available at www.ama-assn.org)

 

 

Privacy and Confidentiality

 

45 CFR 164.506(a) Healthcare entities (Hospitals, Doctors etc.) May share otherwise protected information on patient if purpose is to facilitate care.

 

45 CFR 164.510(b) May share information with family or close friend if this facilitates care and patient does not object.

 

For a good summary of HIPPA go to:

www.hhs.gov/ocr/privacysummary.pdf

 

Treatment of Minors

 

“Informed Consent to the Treatment of Minors” Schlam and Wood, Journal of Law-Medicine Vol. 10 Number 2, Summer 2000 (Case Western Reserve University School of Law)

 

Healthcare Provider’s Right to Follow Their Conscience.

 

Curlin, Lawrence, Chin, Lantos: Religion, Conscience, and Controversial Clinical Practices; N Engl J Med 2007; 356: 593-600

 

      (745 ILCS 70/) Health Care Right of Conscience Act.

 

(745 ILCS 70/2) (from Ch. 111 1/2, par. 5302)

    Sec. 2. Findings and policy. The General Assembly finds and declares that people and organizations hold different beliefs about whether certain health care services are morally acceptable. It is the public policy of the State of Illinois to respect and protect the right of conscience of all persons who refuse to obtain, receive or accept, or who are engaged in, the delivery of, arrangement for, or payment of health care services and medical care whether acting individually, corporately, or in association with other persons; and to prohibit all forms of discrimination, disqualification, coercion, disability or imposition of liability upon such persons or entities by reason of their refusing to act contrary to their conscience or conscientious convictions in refusing to obtain, receive, accept, deliver, pay for, or arrange for the payment of health care services and medical care.

(Source: P.A. 90‑246, eff. 1‑1‑98.)

 

(745 ILCS 70/6) (from Ch. 111 1/2, par. 5306)

    Sec. 6. Duty of physicians and other health care personnel. Nothing in this Act shall relieve a physician from any duty, which may exist under any laws concerning current standards, of normal medical practices and procedures, to inform his or her patient of the patient's condition, prognosis and risks, provided, however, that such physician shall be under no duty to perform, assist, counsel, suggest, recommend, refer or participate in any way in any form of medical practice or health care service that is contrary to his or her conscience.

    Nothing in this Act shall be construed so as to relieve a physician or other health care personnel from obligations under the law of providing emergency medical care.

(Source: P.A. 90‑246, eff. 1‑1‑98.)

 

 

A Simpler Ethical Code:

 

Harry Truman was a different kind of President. He probably made as many important decisions regarding our nation's history as any of the other 42 Presidents. However, a measure of his greatness may rest on what he did after he left the White House.

 

The only asset he had when he died was the house he lived in, which was in Independence Missouri His wife had inherited the house from her mother and other than their years in the White House, they lived their entire lives there.

 

When he retired from office in 1952, his income was a U.S. Army pension reported to have been $13,507.72 a year. Congress, noting that he was paying for his stamps and personally licking them, granted him an 'allowance' and, later, a retroactive pension of $25,000 per year.

 

After President Eisenhower was inaugurated, Harry and Bess drove home to Missouri by themselves. There was no Secret Service following them.

 

When offered corporate positions at large salaries, he declined, stating, "You don't want me. You want the office of the President, and that doesn't belong to me. It belongs to the American people and it's not for sale."

 

Even later, on May 6, 1971, when Congress was preparing to award him the Medal of Honor on his 87th birthday, he refused to accept it, writing, "I don't consider that I have done anything which should be the reason for any award, Congressional or otherwise."

 

As president, he paid for all of his own travel expenses and food.

 

Modern politicians have found a new level of success in cashing in on the Presidency, resulting in untold wealth. Today, many in Congress also have found a way to become quite wealthy while enjoying the fruits of their offices. Political offices are now for sale.

 

Good old Harry Truman was correct when he observed, "My choices in life were either to be a piano player in a whorehouse or a politician. And to tell the truth, there's hardly any difference."

 

WOOD   AAEM

AAEM is an outstanding organization representing the board certified emergency physician.

GARRET-HAUSER    ETHICAL ISSUES

Breaking Bad News:   Ask patient what they are concerned about.  It may help discuss bad news.

Warn the patient that bad news is coming.  Use non-technical terms so patient can understand.  Anticipate the level of their understanding.

ED conversations with patients can veer toward being too blunt due to time constraints.   Be cautious about being too blunt.  Give enough time to be human with the patient or family.

C. Kulstad comment:  When discussing concerning test results like a lung mass on ct, We have an obligation to tell the patient that the most likely diagnosis is cancer based on the test findings.  Other faculty agreed with this.  

Tell patients what the next step is going to be.

Telling family that someone died:  Find out who the family members are in the room.  Get some info from family if you need it before you tell them the person died.   Gotta use the word dead, died or death so families fully understand the irrevocable nature of the situation.

Family requests for non-disclosure of results to the patient:  Ask the family why the request is being made.  Negotiate with the family the best way to handle this situation.  You can ask the patient if they prefer to get the infothemselves or discuss with the family.  You may want to inform families of the standard of truthfulness with patients in this country.   You have a duty to ask patients how they want medical information handled.  If they want the info given to them, you need to give it to them despite the family’s wishes.

Power of Attorney gives the surrogate the same decisional capacity as the patient would normally have.  There cannot be 2 powers of attorney.  It can only be one person.  There can be a successor named but they cannot make decisions if the power of attorney is present.

Surrogate act allows spouse, adult children or moving down the hierarchy, other family members or close friends to make decisions for a patient with a qualifying condition who doesn’t have decisional capacity.   

To fill out the form to withdraw care (like taking out the ET tube or taking out feeding tube) you need two docs to sign off the WITHDRAWL OF CARE FORM  that patient has a terminal condition.

Harwood comment: Utilize ethics committee to help with management of difficult clinical decisions when there is some time.  Also when breaking news to a family that a patient has died he prefers tell them the patient has died very early in the conversation.

KUTKA  M AND M TRAUMA

21yo male shot in buttock.

Get prepared prior to patient arrival even if the EMS report on the radio says “stable vitals”

DDX of Combative behavior: guy is a jerk, intoxication, hypoperfusion.    Assume hypoperfusion until proven otherwise.

Even if bleeding seems venous don’t downplay the volume or significance of the blood loss.

Even if it appears to be a “simple” trauma don’t treat it like that.  Treat aggressively and discuss your concerns with attending.

When giving blood products in a bleeding patient be sure to give enough prbcs AND ffp and platelets.  Follow the massive transfusion protocol.

Criteria for Massive Transfusion: Penetrating mechanism, positive fast, arrival BP<90, arrival HR>120.  2criteria=40% chance and 4criteria=100% chance of needing massive transfusion.

When breaking bad news: discuss with Chaplain, have security with you, limit the # of family members in the quiet room, make sure you have an exit strategy.

Barounis comment: If you order the massive transfusion protocol and don’t use all the blood, it will be sent back to the blood bank to be used again. 

 

Conference Notes 7-24-2012

Conference Notes 7-24-2012

WILLIAMSON  STUDY GUIDE  RESUSCITATION QUESTION SLAM

Junctional rhythm does not occur in healthy hearts so you have to investigate for ischemia or other pathology.

Brugada Syndrome is a genetic condtion that affects phase 0 sodium channels.  Causes sudden death in structurally normal hearts.   Affects asian men more commonly.   EKG demonstrates RBBB with j point elevation and widening in leads V1-3.   

Elise comment:  not everything that looks like Brugada is Brugada so you need cardiology to consult on these EKG’s if there is a question.

Treat WPW with afib with procainamide if stable.  If unstable, cardiovert. 

Norepi is a good pressor to use if you don’t want to increase tachycardia too much.  Alpha effects (vasoconstriction) are more prominent.

When resuscitating a patient with septic shock, vasopressors work best with a full vascular bed.   Pressors can falsely elevate cvp so be sure you are giving adequate fluids.   There is no evidence that trandelenburg position improves pt outcome or cardiopulmonary function.  There is some evidence it may worsen cv status.  So don’t use it.

For massive transfusions, pt’s should receive prbc’s/platelets/ffp in 1:1:1 ratio.

For pelvic or abdominal injuries  you want iv access above the diaphragm.  IO in the proximal humerus is a great second line access means is you can’t get peripheral iv access.

Anaphylaxis is an IGE-mediated immune reaction.   IM Epinephrine in the thigh (0.3-0.5mg) is the treatment for adults.  If you have two of the following: skin/respiratory/gi/cardiovascular symptoms you have anaphylaxis.  

Transvenous pacing for an unstable patient with bradycardia or heart block is indicated if transcutaneous pacing at max output is not getting good capture.  In the meantime try atropine. 

Barounis comment: Check for  hyperkalemia if transcutaneous pacing is unsuccessful.

Initial management of rapid afib is rate control with cardizem.

Barounis comment: I would prefer elective cardioversion for acute rapid afib.  Elise comment: totally would want to be shocked for rapid afib of less than 48 hours.

BAROUNIS/GRIPPO    INTRO TO CODE 44

SORRY I MISSED THIS LECTURE

KETTANEH  INTRO TO SEPSIS

SIRS: hr>90, tachypnea, elevated wbc, abnormal temp either hi or low

Sepsis is SIRS plus an infection

Severe sepsis is sepsis with hypotension prior to IV fluids

Septic Shock: severe sepsis not responding to fluid bolus of 20cc/kg

 Sepsis patients need at least 20ml/kg of NS with a minimum of 1 liter over 30 minutes.

Broad spectrum in the first hour improves mortality.  Delay of antibiotics increases mortality of about 7% per hour.

Early Goal Directed Therapy: has been shown to reduce sepsis mortality compared to standard care.  Indication of EGDT is persistent hypotension or lactate >4 despite initial fluids.  Place a CVP line above the diaphragm.  CVP is a proxy for preload.  CVP helps you to optimize volume status.  Goal is CVP of 8-12 or 12-15 if the patient is on a ventilator.  If CVP less than 8 give more fluids.   Next step is to start vasopressors.  Vasopressors do not improve mortality.  No data to say which pressor is superior.   Norepi however is the drug of choice due to less adverse effects.    Central venous oxygen satuation is a marker of success/mortality.  So get a VBG off your central line.   If Scv02 is <70% you got problems.   Intubate/Check hgb/start dobutamine as an inotrope.  Transfuse if hgb less than  7. 7-10 is a grey zone.  Lactate  clearance is a reasonable proxy for Scvo2.  

No steroid use/not indicated for sepsis. Steroids only if pt’s on chronic steroids and are steroid dependent.   Procalcitonin if nl may be a sign you can stop antibiotics but evidence is weak.

Wise comment: if Scvo2 is low, early intubation with neuromuscular blockade is indicated.

C Kulstad: Sepsis patients ned high fluid maintenance rates like 200ml/hr

Lovell comment:  If you give fluid bolus to patients give a liter not 200ml bolus.  Also use EGDT protocol in our EMR.

Hermann comment:  IV infusion pumps do not get the fluids in fast enough.  You have to hang fluids with a pressure bag and no pump.   Corroborated by Dr. E Kulstad/Lovell

Harwood comment: Questioned that pressors in septic shock don’t improve mortality.

MCDERMOTT    SEVERE COPD EXACERBATIONS

Definition: increase in symptoms of cough/sputum/dyspnea.  All copdr’s  have a decreased FEV1/FVC ratio.

Treatment priority is to ensure oxygenation.   Hypercapnea is well tolerated.    Get O2 sat to 92% and 95% in dark skinned patients.   Can use venti masks but usually 4L pnc is enough to correct oxygenation problems in copdr’s.  If you need more oxygen support you gotta think about other things.

Bipap has been shown to decrease need for intubation, decrease risk of nosocomial infection, and decrease mortality.    Intial settings are 8-12 inspiratory and 3-5 expiratory.  If you need to adjust settings a reasonable move would be 15/7.

Vent management for intubated patients: high inspiratory flow rates and low tidal volumes (5-7ml/kg) and low rates.  This will help to minimize airway pressures.

Treatment: Nebs are more effective than in mdi’s in COPD exacerbation.   Magnesium can be used.  Theophylline has not been shown to improve outcomes in COPD.  Chest PT also not indicated in acute COPD exacerbation because it can worsen an acute exacerbation.   Steroids have been shown to improve symptoms, lung function and decrease hospital stay.  About a third of COPD exacerbations are triggered by bacterial infection so antibiotics are indicated for acute exacerbations.  Sputum cultures are not useful.   Pseudomonas risk factors:4 episodes/year, recent hospitalization, previous pseudomonas, or severe copd.   If risk factors for pseudomonas are present give zosyn, if not then ceftriaxone.    

Most common causes of death in an admitted COPD patient: heart failure, pneumonia, PE, then COPD.

Work UP: CXR, EKG, Troponin, CBC, BMP,  ABG,  +/- Blood cultures.

Girzadas comment: What triggers PE work up (one study showed 20% incidence of pe in COPD exacerbation but my experience does not match that)?    Robbie: persistent hypoxia or remarkable risk factors,  exam findings, lab or cxr findings suggestive of PE.

Harwood comment:   CO2 narcosis is very common due to excess O2 administration.  Beware of giving too much oxygen to COPDR’s.   If the patient needs to be intubated for CO2 narcosis, they likely will be hard to get off the ventilator.   Try bipap first to reduce CO2 retention but for very high PCO2’s it may not work.

PUTMAN SEVERE CHF EXACERBATION

  Common ED clinical conundrum is separating out pneumonia/chf/copd.

Bipap has been shows to decrease intubations/admissions/mortality.   PEEP helps.  12/5 again is a reasonable starting point for the bipap machine.

Nitroglycerine (both IV and Sublingual) is the key medication treatment.   Give 4 sprays of ntg to give a 1600microgram bolus rapidly.   Start 50mcg/min drip and titrate up rapidly to around 200 mcg/min.  If patient still not improving and BP still up, you can add nitroprusside drip.

Beckemeyer and Harwood comments: Look for improvement from ntg by improved dyspnea and improved blood pressure.   You will only be at the very high levels of ntg administration for a short while because BP will start going down.  You will need to be alert as to when to start going down on ntg dose.

Diuretics alone can increase mortality. Diuretics can worsen renal function.  You need to not give lasix until you have ntg going.

ACE-I ‘s have been shown to improve patient outcomes.

Contraindications to  NTG vasodilatation:  RV infarct, aortic stenosis, hypertrophic cardiomyopathy, hypotension.

Barounis comment: how do you manage patient pulling off bipap mask?   Elise and Christine say give ativan cautiously.   If they either keep pulling of mask or become too somnolent you proceed to intubation.

Girzadas comment: Be alittle bit patient for the NTG to work for the first 15 minutes.  Most patients don’t need to be intubated.  The nitro will work magic if given alittle time.

BAROUNIS   ALTERED MENTAL STATUS

SORRY I MISSED THIS LECTURE